ENADE Comentado 2008: Física

Propaganda
ENADE COMENTADO 2008
Física
Chanceler
Dom Dadeus Grings
Reitor
Joaquim Clotet
Vice-Reitor
Evilázio Teixeira
Conselho Editorial
Ana Maria Lisboa de Mello
Elaine Turk Faria
Érico João Hammes
Gilberto Keller de Andrade
Helenita Rosa Franco
Jane Rita Caetano da Silveira
Jerônimo Carlos Santos Braga
Jorge Campos da Costa
Jorge Luis Nicolas Audy – Presidente
José Antônio Poli de Figueiredo
Jurandir Malerba
Lauro Kopper Filho
Luciano Klöckner
Maria Lúcia Tiellet Nunes
Marília Costa Morosini
Marlise Araújo dos Santos
Renato Tetelbom Stein
René Ernaini Gertz
Ruth Maria Chittó Gauer
EDIPUCRS
Jerônimo Carlos Santos Braga – Diretor
Jorge Campos da Costa – Editor-chefe
Maria Eulália Pinto Tarragó
Délcio Basso
(Organizadores)
ENADE COMENTADO 2008
Física
Porto Alegre
2011
© EDIPUCRS, 2011
CAPA Rodrigo Valls
REVISÃO DE TEXTO Rafael Saraiva
EDITORAÇÃO ELETRÔNICA Gabriela Viale Pereira
Questões retiradas da prova do ENADE 2008 da Física
EDIPUCRS – Editora Universitária da PUCRS
Av. Ipiranga, 6681 – Prédio 33
Caixa Postal 1429 – CEP 90619-900
Porto Alegre – RS – Brasil
Fone/fax: (51) 3320 3711
e-mail: [email protected] - www.pucrs.br/edipucrs
Dados Internacionais de Catalogação na Publicação (CIP)
E56
ENADE comentado 2008 : física [recurso eletrônico] /
organizadores, Maria Eulália Pinto Tarragó, Délcio
Basso. – Dados eletrônicos. – Porto Alegre :
EDIPUCRS, 2011.
92 p.
Sistema requerido: Adobe Acrobat Reader
Modo de Acesso: <http://www.pucrs.br/edipucrs/>
ISBN 978-85-397-0083-7 (on-line)
1. Ensino Superior – Brasil – Avaliação. 2. Exame
Nacional de Desempenho de Estudantes. 3. Física –
Ensino Superior. I. Tarragó, Maria Eulália Pinto. II. Basso,
Délcio.
CDD 378.81
Ficha Catalográfica elaborada pelo Setor de Tratamento da Informação da BC-PUCRS.
SUMÁRIO
APRESENTAÇÃO ..................................................................................................... 8
Ana Maria Marques da Silva
COMPONENTE ESPECÍFICO - NÚCLEO COMUM
QUESTÃO 11 ........................................................................................................... 12
Artur Majolo Scheid, Maria Eulália Pinto Tarragó e Délcio Basso
QUESTÃO 12 ........................................................................................................... 13
Délcio Basso e Maria Eulália Pinto Tarragó
QUESTÃO 13 ........................................................................................................... 15
Artur Majolo Scheid, Maria Eulália Pinto Tarragó e Délcio Basso
QUESTÃO 14 ........................................................................................................... 17
Maria Eulália Pinto Tarragó e Délcio Basso
QUESTÃO 15 ........................................................................................................... 18
Maria Eulália Pinto Tarragó e Délcio Basso
QUESTÃO 16 ........................................................................................................... 19
Rafael L. Zimmer
QUESTÃO 17 ........................................................................................................... 22
Délcio Basso e Maria Eulália Pinto Tarragó
QUESTÃO 18 ........................................................................................................... 23
Natthan Ruschel Soares, Maria Eulália Pinto Tarragó e Délcio Basso
QUESTÃO 19 ........................................................................................................... 25
Délcio Basso
QUESTÃO 20 ........................................................................................................... 27
Elias Cantarelli Hoffmann, Maria Eulália Pinto Tarragó e Délcio Basso
QUESTÃO 21 ........................................................................................................... 28
Elias Cantarelli Hoffmann, Maria Eulália Pinto Tarragó e Délcio Basso
QUESTÃO 22 ........................................................................................................... 30
Artur Majolo Scheid, Maria Eulália Pinto Tarragó e Délcio Basso
QUESTÃO 23 ........................................................................................................... 32
Elaine Evani Streck
QUESTÃO 24 ........................................................................................................... 34
Natthan Ruschel Soares, Maria Eulália Tarragó e Délcio Basso
QUESTÃO 25 ........................................................................................................... 35
Juliane Bernardes Marcolino, Maria Eulália Tarragó e Délcio Basso
QUESTÃO 26 ........................................................................................................... 37
Elaine Evani Streck e Janaína Galho Borges
QUESTÃO 27 ........................................................................................................... 39
Aldoir Rigoni
QUESTÃO 28 ........................................................................................................... 40
Délcio Basso
QUESTÃO 29 ........................................................................................................... 42
Elaine Evani Streck e Janaína Galho Borges
QUESTÃO 30 - DISCURSIVA.................................................................................. 44
Alexandre Ferret, Maiara Oliveira Dalenogare, Márcio Galhardi, Maria Eulália
Pinto Tarragó e Délcio Basso
COMPONENTE ESPECÍFICO - BACHARELADO
QUESTÃO 31 ........................................................................................................... 47
Maria do Carmo Baptista Lagreca e Ricardo Meurer Papaléo
QUESTÃO 32 ........................................................................................................... 50
Elias Cantarelli Hoffmann, Maria Eulália Pinto Tarragó e Délcio Basso
QUESTÃO 33 ........................................................................................................... 52
Maria Eulália Pinto Tarragó e Délcio Basso
QUESTÃO 34 - QUESTÃO ANULADA ................................................................... 54
Adriano Moehlecke, Izete Zanesco e Aline Cristiane Pan
QUESTÃO 35 ........................................................................................................... 56
Cássio Stein Moura
QUESTÃO 36 ........................................................................................................... 58
Délcio Basso e Maria Eulália Pinto Tarragó
QUESTÃO 37 ........................................................................................................... 60
Maria Eulália Pinto Tarragó e Délcio Basso
QUESTÃO 38 ........................................................................................................... 62
Cássio Stein Moura
QUESTÃO 39 - DISCURSIVA.................................................................................. 64
Sayonara Salvador Cabral da Costa
QUESTÃO 40 - DISCURSIVA.................................................................................. 66
Cássio Stein Moura
COMPONENTE ESPECÍFICO - LICENCIATURA
QUESTÃO 41 ........................................................................................................... 71
João Bernardes da Rocha Filho
QUESTÃO 42 ........................................................................................................... 73
João Bernardes da Rocha Filho
QUESTÃO 43 ........................................................................................................... 75
João Bernardes da Rocha Filho
QUESTÃO 44 ........................................................................................................... 77
João Bernardes da Rocha Filho
QUESTÃO 45 ........................................................................................................... 79
João Bernardes da Rocha Filho
QUESTÃO 46 - QUESTÃO ANULADA ................................................................... 81
Aldoir Rigoni
QUESTÃO 47 ........................................................................................................... 83
Aldoir Rigoni
QUESTÃO 48 ........................................................................................................... 84
Aldoir Rigoni
QUESTÃO 49 - DISCURSIVA.................................................................................. 86
Maria Eulália Pinto Tarragó e Délcio Basso
QUESTÃO 50 - DISCURSIVA.................................................................................. 88
Adriana Schier e Sayonara Cabral da Costa
LISTA DE CONTRIBUINTES ................................................................................... 91
APRESENTAÇÃO
No ensino superior, as avaliações institucionais possibilitam a análise de
diversas dimensões do ensino, da pesquisa e da extensão, evidenciando objetivos e
compromissos das instituições e de seus cursos de graduação.
O Sistema Nacional de Avaliação da Educação Superior (Sinaes), instituído
através da Lei no 10.861, de 14/04/2004, tem como objetivo assegurar o processo
continuado de avaliação das instituições de educação superior, dos cursos de
graduação e do desempenho dos estudantes.
Como forma de atingir as instituições de ensino superior em sua totalidade, o
Sinaes possui três componentes principais: a avaliação interna e externa das
instituições, a avaliação dos cursos e a avaliação do desempenho dos estudantes,
por meio do Exame Nacional de Desempenho dos Estudantes (ENADE).
O ENADE é constituído por um questionário socioeconômico e uma prova. Por
meio do questionário socioeconômico, é possível compor o perfil dos estudantes,
integrando informações do seu contexto às suas percepções sobre a instituição de
ensino e suas vivências.
A prova busca avaliar como o estudante é capaz de utilizar suas competências e
habilidades, assim como analisar sua evolução, por meio da avaliação em dois
momentos: no primeiro e no último ano da graduação. Além das competências
profissionais, a formação geral e a abordagem dos temas transversais são avaliados,
enriquecendo o sistema de avaliação e agregando-lhe elementos de reflexão.
As questões da prova, de natureza objetiva e discursiva, priorizam temas
contextualizados e atuais, problematizados na forma de situações problemas e
estudos de caso, propiciando respostas por meio de múltipla escolha.
A prova compõe-se de duas partes: Formação Geral e Componente Específico.
A parte da Formação Geral é comum às provas aplicadas a todos os cursos
participantes
do
ENADE
daquela
edição.
Tem
como
objetivo
investigar
competências, habilidades e conhecimentos gerais dos estudantes, analisando a
compreensão de temas relacionados à realidade nacional e internacional, que não
pertencem necessariamente ao âmbito específico da profissão.
8
Maria Eulália Tarragó e Délcio Basso (Orgs.)
A parte do Componente Específico contempla as especificidades de cada
curso, tanto no domínio dos conhecimentos quanto nas habilidades esperadas para
cada perfil profissional. Tem como objetivo investigar os conteúdos do curso, por
meio de níveis distintos de habilidades e saberes.
Para o curso de Física, o Componente Específico da prova do ENADE 2008 foi
dividido em duas partes, sendo uma parte denominada Núcleo Comum (questões 11
a 30), realizada por todos os estudantes de Física, outra parte para avaliação do
componente específico do Bacharelado (questões 31 a 40) e outra para o
componente específico da Licenciatura (questões 41 a 50).
Na parte de avaliação do Núcleo Comum foram apresentadas 20 (vinte)
questões que se referiam aos conteúdos gerais da formação em Física, sendo 1
(uma) questão discursiva e 19 (dezenove) questões de múltipla escolha. Para a
avaliação do componente específico do Bacharelado em Física havia 2 (duas)
questões discursivas e 8 (oito) questões objetivas. Da mesma forma, o componente
específico da Licenciatura em Física foi avaliado por meio de 2 (duas) questões
discursivas e 8 (oito) questões de múltipla escolha.
Esta publicação dá continuidade a Série ENADE Comentado, publicada pela
EDIPUCRS, que apresenta a resolução comentada das provas de diferentes cursos,
desde as edições de 2004 do ENADE. O formato eletrônico da coleção permite que
os interessados tenham acesso universal e facilitado às publicações.
O ENADE Comentado 2008: Física, ora apresentado, disponível gratuitamente
na Internet, apresenta a resolução comentada de todas as 40 (quarenta) questões do
componente específico das provas aplicadas aos estudantes do curso de Física, tanto
em relação aos conteúdos gerais (Núcleo Comum), quanto aos componentes
específicos do Bacharelado e da Licenciatura. Na publicação estão incluídos,
inclusive, os comentários das questões que foram posteriormente anuladas pelo MEC.
Esta publicação é o resultado concreto de um processo de reflexão realizado
por professores e estudantes do Curso de Física da PUCRS, a partir da realização
do ENADE 2008. Para comemorar os bons resultados obtidos pelos alunos do Curso
de Física da PUCRS, no final de 2009, foi organizado um evento, no qual estudantes
concluintes e professores foram convidados a apresentarem suas percepções sobre
a prova e comentarem algumas questões. Nesta ocasião, na qual estiveram
ENADE Comentado 2008: Física
9
presentes professores da Faculdade de Física e estudantes do curso, foi realizado
um convite a todos que desejassem colaborar na publicação das questões
comentadas do ENADE Física. Os estudantes, entusiasmados com a possibilidade
de uma produção conjunta, procuraram a orientação de professores para a
elaboração da resolução comentada de questões de sua escolha. Estamos certos de
que esta experiência produziu um impacto positivo na formação e amadurecimento
dos estudantes que aceitaram tal desafio, assim como uma aproximação e
comprometimento de professores e estudantes em relação à importância desta
avaliação.
Além da materialização da profícua parceria entre docentes e discentes da
Faculdade de Física, esta publicação pretende representar um subsídio de estudo e
consulta para estudantes e professores, possibilitando a discussão e resolução de
problemas em diferentes tópicos de Física.
A organização desta publicação foi realizada pelos professores Délcio Basso e
Maria Eulália Pinto Tarragó, que assumiram esse desafio e souberam motivar o
grupo para a produção conjunta, realizando, além da resolução de algumas
questões, a revisão e uniformização de todo o material.
O trabalho cooperativo para a publicação do ENADE Comentado 2008: Física
envolveu um total de 13 professores da Faculdade de Física e 9 estudantes e
diplomados do Curso de Física da PUCRS.
Aos organizadores e a todos os colaboradores desta publicação, agradecemos
pela dedicação e competência com que realizaram este trabalho aqui apresentado.
Porto Alegre, maio de 2011
Ana Maria Marques da Silva
Diretora da Faculdade de Física/PUCRS
10
Maria Eulália Tarragó e Délcio Basso (Orgs.)
COMPONENTE ESPECÍFICO
NÚCLEO COMUM
QUESTÃO 11
No dia 19 de agosto de 2008 foi lançado, pelo foguete russo Proton Breeze M o I4F3, um dos maiores satélites já construídos, que será utilizado para serviços de
telefonia e Internet. O conjunto foguete + satélite partiu de uma posição vertical.
Sendo a massa m do satélite igual a 6 toneladas, a massa M do foguete igual a 690
toneladas e a velocidade de escape dos gases no foguete (vgases) igual a 1.500 m/s,
qual é a quantidade mínima de gás expelida por segundo (∆mgases /∆t) para que o
foguete eleve o conjunto no instante do lançamento?
(Considere g = 10 m/s2)
(A)
9,3 x 103 kg/s
(B)
4,6 x 103 kg/s
(C) 2,3 x 103 kg/s
(D) 2,3 x 102 kg/s
(E)
2,2 x 104 kg/s
Gabarito: B
Tipo de questão: Escolha simples, com indicação da alternativa correta.
Resolutores: Artur Majolo Scheid, Profª. Dr. Maria Eulália Pinto Tarragó e Prof. Me.
Délcio Basso
Comentário:
Para resolver essa questão vamos lembrar o Princípio de Conservação da
Quantidade de Movimento de acordo com o surgimento de forças internas em um
sistema não altera a quantidade de movimento do mesmo.
Nessa questão o nosso sistema é constituído pelo foguete com o satélite mais
os gases que estão sendo expelidos. Portanto, a força mínima exercida pelos gases
expelidos deve ser igual, em módulo, ao peso do foguete com o satélite.
Logo, ( M + m) g =
pelos
∆m gases
∆t
seus
v gases . Isolando
∆m gases
valores
e substituindo as grandezas
∆t
numéricos
obtemos
3
m
( M + m) g (690 × 10 + 6 × 10 )kg × 10 s 2
=
=
= 4,6 × 10 3 kg .
s
m
∆t
v gases
1500
s
resposta certa é a alternativa (B).
∆m gases
12
3
Maria Eulália Pinto Tarragó e Délcio Basso (Orgs.)
Portanto,
a
QUESTÃO 12
A figura abaixo representa o movimento de uma bola, em um plano vertical,
registrado com uma fonte de luz pulsada a 20 Hz.
(As escalas vertical e horizontal são iguais.)
Supondo que a aceleração da gravidade local seja igual a 10 m/s2, qual é o módulo
da componente horizontal da velocidade da bola?
(A) 2 m/s.
(B) 3 m/s
(C) 4 m/s
(D) 5 m/s
(E) 6 m/s
Gabarito: A
Tipo de questão: Escolha associada com indicação da resposta correta.
Resolutores: Prof. Me. Délcio Basso e Profª. Dr. Maria Eulália Pinto Tarragó
Comentário:
∆x
,
∆t
portanto precisamos descobrir a distância ∆x percorrida num dado intervalo de tempo ∆t.
O módulo da velocidade horizontal da bola é constante e igual a vx =
ENADE Comentado 2008: Física
13
A análise da figura permite escrever que a bola leva
∆t =
o tempo
1
× 10s = 0,5s para descer desde o ponto mais alto de sua trajetória, em que sua
20
velocidade na direção vertical v y é nula, até a última representação da bola,
percorrendo uma distância na vertical aproximadamente igual a ∆y = −12 unidades.
Por
outro
lado,
∆y = −12unidades = −
a
relação
cinemática
nos
diz
que
g 2
10
t = − m / s 2 (0,5s ) 2 = −1,25m. Portanto, cada unidade equivale
2
2
a 0,104m. Sabemos que enquanto a bola cai, ela percorre 10 unidades na direção
horizontal,
vx =
14
portanto,
o
módulo
da
velocidade
horizontal
∆x 10 × 0,104m
=
= 2m / s , sendo a opção correta a alternativa (A).
∆t
0,5s
Maria Eulália Pinto Tarragó e Délcio Basso (Orgs.)
da
bola
será
QUESTÃO 13
Uma brincadeira de criança que mora perto de um riacho é atravessá-lo usando uma
corda,
essa
concretizar.
travessia
Para
avaliar
pode
o
não
se
perigo
da
TIPLER, P.A.; MOSCA, G. Física para cientistas e
engenheiros, V1 - Mecânica, Oscilações, Ondas,
Termodinâmica. Rio de Janeiro: LTC, 2006.
corda amarrada a uma árvore perto da margem. Dependendo da resistência da
travessia, pode-se usar como modelo o
movimento do pêndulo, e calcular a tensão
máxima
que
a
corda
pode
suportar.
Considerando que a corda faz, inicialmente,
um ângulo de 60° com a vertical, qual é a
tensão máxima a ser suportada pela corda
para que uma criança de 30 kg atravesse o
riacho?
(Considere g = 10 m/s2)
(A) 200 N
(B) 300 N
(C) 600 N
(D) 900 N
(E) 1.200 N
Gabarito: C
Tipo de questão: Escolha simples com indicação da resposta correta.
Resolutores: Artur Majolo Scheid, Profª. Dr. Maria Eulália Pinto Tarragó e Prof. Me.
Délcio Basso
Comentário:
O menino pendurado na corda está submetido a uma força de tensão e a sua
força peso. A tensão máxima Tmax ocorre no ponto mais baixo da trajetória, em que a
força peso mg, para baixo, e a tensão, para cima, estão na mesma direção. Por
outro lado, como a trajetória descrita pelo menino é um arco de circunferência, a
força resultante sobre o menino, no ponto mais baixo da sua trajetória, é a força
centrípeta dada por Fc=Tmax−mg.
ENADE Comentado 2008: Física
15
mv 2
Isolando Tmax e lembrando que Fc = mac =
onde m a massa do menino, v
r
é a sua velocidade no ponto mais baixo da trajetória e r é raio da trajetória (igual ao
mv 2
comprimento da corda), teremos Tmax =
+ mg .
r
Precisamos, então, descobrir quanto vale v2.
Para tanto, vamos considerar a conservação da
energia mecânica que nos permite escrever a
igualdade mgh =
mv 2
2
ou v 2 = 2 gh . A altura h é
obtida da relação trigonométrica mostrada na figura
ao
lado,
em
que
h(1 − 0,5) =
r
; h = r − r cos 60 .
2
Portanto v 2 = gr .
Substituindo a relação anterior na expressão
de Tmax obtemos:
Tmáx =
mgr
+ mg = 2mg = 600 N
r
.
Portanto, a alternativa correta é a (C).
16
Maria Eulália Pinto Tarragó e Délcio Basso (Orgs.)
QUESTÃO 14
Um disco gira livremente, com velocidade angular ω, em torno de um eixo vertical
que passa pelo seu centro. O momento de inércia do disco em relação ao eixo é I1.
Um segundo disco, inicialmente sem rotação, é colocado no mesmo eixo e cai sobre
o primeiro disco, como mostra a figura. Após algum tempo, o atrito faz com que os
dois discos girem juntos. Se o momento de inércia do segundo disco é I2, qual é a
velocidade angular final de rotação do conjunto?
(A)
(B)
(C)
(D)
(E)
Gabarito: D
Tipo de questão: Escolha simples com indicação da resposta correta.
Resolutores: Profª.Dr. Maria Eulália Pinto Tarragó e Prof. Me. Délcio Basso
Comentário:
O sistema é formado pelos dois discos, um já em rotação e pelo outro, sem
rotação, que cai sobre o primeiro. Assim, na ausência de torques externos, deve
ocorrer a conservação do momento angular: Li = I 1ω = L f = ( I 1 + I 2 )ω f .
Então, ω f = ω
I1
. Portanto, a alternativa correta é a (D).
I1 + I 2
ENADE Comentado 2008: Física
17
QUESTÃO 15
Uma jovem mãe prepara o banho para o seu bebê. Ela sabe que a temperatura da
água da torneira é de 20 °C, e que a temperatura ideal da água para o banho é de
36 °C. Quantos litros de água fervendo a mãe deve misturar com a água da torneira
para obter 10 litros de água na temperatura ideal para o banho?
(A)
2,5
(B)
2,0
(C) 1,5
(D) 1,0
(E)
0,5
Gabarito: B
Tipo de questão: Escolha simples com indicação da resposta correta.
Resolutores: Profª.Dr. Maria Eulália Pinto Tarragó e Prof. Me. Délcio Basso
Comentário:
Para resolver essa questão devemos lembrar que a soma do calor ganho pela
água da torneira com o calor perdido pela água fervendo é nula, isto é, Q g + Q p = 0 .
Logo, m f c∆T f + mq c∆Tq = 0 , em que mf é a massa de água fria e ∆T f é a variação de
sua temperatura; semelhantemente mq é a massa de água quente e ∆Tq é a variação
de sua temperatura. Lembrando que o calor específico da água é 1,0cal/g°C e que a
temperatura da água fervendo é 100°C, a 1atm, então a equação anterior, fica:
m f (36 − 20) + mq (36 − 100) = 0 , resultando em 16m f − 64mq = 0 (Eq.A).
Por outro lado, sabemos que a soma das massas de água fervendo (água
quente) e de água da torneira (água fria) é 10kg, uma vez que a massa específica
da água é 1,0kg/litro. Logo, podemos escrever m q + m f = 10kg (Eq.B). Substituindo a
equação B na equação A encontramos que a massa da água quente é 2,0kg,
correspondendo a um volume de 2,0litros, o que nos permite marcar como correta a
alternativa (B).
18
Maria Eulália Pinto Tarragó e Délcio Basso (Orgs.)
QUESTÃO 16
Cinco sensores foram utilizados para medir a temperatura de um determinado corpo.
As curvas de calibração da resistência elétrica, em função da temperatura destes
sensores, são apresentadas no gráfico abaixo.
(1) Germânio
(2) Vidro-Carbono
(3) Platina
(4) Cernox 
(5) Rox 
Analisando-se o gráfico, foram feitas as afirmativas a seguir.
I - O sensor (2) só deve ser utilizado para temperaturas superiores a 20 K.
II - Para temperaturas entre 1 K e 3 K apenas o sensor (5) pode ser utilizado.
III - Quando a resistência do sensor (1) atingir o valor de cerca de 7Ω, o sensor (4)
estará com uma resistência um pouco superior a 2 kΩ.
IV - O sensor (3) é o único a ser empregado para temperaturas na faixa de 20 K a
300 K.
São verdadeiras APENAS as afirmações
(A) I e II
(B) I e IV
(C) II e III
(D) II e IV
(E) III e IV
ENADE Comentado 2008: Física
19
Gabarito: C
Tipo de questão: Escolha combinada com indicação da alternativa correta.
Resolutor: Rafael L. Zimmer – Aluno Concluinte
Comentário:
A afirmativa I é falsa, pois o sensor (2) pode ser utilizado na faixa de
temperaturas entre 4K até 300K.
A afirmativa II é verdadeira, pois “Para temperaturas entre 1K e 3K apenas o
sensor (5) pode ser utilizado”.
20
Maria Eulália Pinto Tarragó e Délcio Basso (Orgs.)
A afirmativa III é verdadeira, pois “quando a resistência do sensor (1) atingir o
valor de cerca de 7Ω, o sensor (4) estará com uma resistência um pouco superior a
2kΩ”.
A afirmativa IV é falsa, pois os sensores 2 e 4 podem também ser
empregados na faixa de temperaturas entre 20 K e 300 K.
Portanto, a alternativa correta é a (C).
ENADE Comentado 2008: Física
21
QUESTÃO 17
Uma certa quantidade de um gás ideal ocupa um volume inicial Vi à pressão pi e
temperatura Ti. O gás se expande até o volume Vf (Vf > Vi), segundo dois processos
distintos: (1) a temperatura constante e (2) adiabaticamente.
Com relação à quantidade de calor Q fornecida, ao trabalho W realizado e à
variação de energia interna ∆E de cada processo, pode-se afirmar que
I - Q1= Q2
II - Q1 > Q2
III - ∆E1 = ∆E2
IV - ∆E1 < ∆E2
V - W1 > W 2
São verdadeiras APENAS as afirmações
(A) I e III
(B) I e IV
(C) II e V
(D) III e V
(E) IV e V
Gabarito: C
Tipo de questão: Escolha combinada com indicação da resposta correta.
Resolutores: Prof. Me. Délcio Basso e Profª. Dr. Maria Eulália Pinto Tarragó
Comentário:
A variação da energia interna do gás para o processo isotérmico (1) é nula,
V
∆E1 = 0 , logo Q1=W 1= nRTi ln( f ) , como V f > Vi então Q1=W 1>0.
Vi
A variação da energia interna do gás para o processo adiabático (2) é igual ao
negativo do trabalho, ∆E2 = −W2 , uma vez que o calor envolvido nesse processo é
nulo, Q2=0. Como W 2>0, então ∆E2 <0.
Como o trabalho W é equivalente a área sob a curva no diagrama PV,
pressão em função de volume, é fácil verificar que W 1>W 2, pois no processo 2 o gás
esfria na medida que expande, portanto T2<T1.
Logo, somente as afirmativas II (Q1>Q2) e V (W 1>W 2) são verdadeiras,
correspondendo à opção (C).
22
Maria Eulália Pinto Tarragó e Délcio Basso (Orgs.)
QUESTÃO 18
Em 1816, o escocês Robert Stirling criou uma máquina térmica a ar quente que
podia converter em trabalho boa parte da energia liberada pela combustão externa
de matéria-prima. Numa situação idealizada, o ar é tratado como um gás ideal com
calor específico molar Cv = 5 R/2, onde R é a constante universal dos gases. A
máquina idealizada por Stirling é representada pelo diagrama P versus V da figura
abaixo. Na etapa C → D (isotérmica), a máquina interage com o reservatório quente,
e na etapa A → B (também isotérmica), com o reservatório frio. O calor liberado na
etapa isovolumétrica D → A é
recuperado integralmente na
etapa
B
→ C,
também
isovolumétrica. São conhecidas
as temperaturas das isotermas
T1 e T2, os volumes VA e VB e o
número de moles n de ar
contido na máquina.
HALLIDAY, D; RESNICK, R; WALKER, J.
Fundamentos de Física, v.2, 4 ed. Rio
de Janeiro: LTC, 1996.
Qual o rendimento do ciclo e sua variação total de entropia?
(A)
(B)
(C)
(D)
(E)
ENADE Comentado 2008: Física
23
Gabarito: D
Tipo de questão: Escolha simples com indicação da resposta correta.
Resolutores: Natthan Ruschel Soares, Profª. Dr. Maria Eulália Pinto Tarragó e Prof.
Me. Délcio Basso
Comentário:
Para resolver essa questão o estudante deve lembrar que o rendimento
W
termodinâmico de um motor é dado por ε = res , em que Wres é o trabalho resultante
Q for
no ciclo e Qfor é o calor fornecido.
O trabalho resultante é dado por WAB + W CD, uma vez que os trabalhos W BC e
WDA são nulos, pois corresponde a etapas isométricas.
Para um processo isotérmico o trabalho realizado é W = nRT ln
Vf
Vi
que é igual
ao calor trocado.
No enunciado afirma-se que o calor liberado na etapa isovolumétrica de D
para A, QDA, é recuperado integralmente na etapa isovolumétrica de B para C, QBC,
assim o calor fornecido no ciclo é Qfor=W CD= nRT2 ln
Portanto,
a
eficiência
do
motor
VD
.
VC
ε=
será
Wres W AB + WCD
W
= 1 + AB .
=
Q for
WCD
WCD
Reescrevendo:
ε = 1+
nRT1 ln(VB
nRT2 ln(VD
VA
VC
)
)
. Porém, (V B
VA
V
)=( C
VD
V
) , assim ln( B
VA
V
) = − ln( D
VC
),
T1
.
T2
Como o ciclo é formado por transformações reversíveis, a variação resultante
então ε = 1 −
da entropia é nula. Logo, a alternativa correta é a (D).
24
Maria Eulália Pinto Tarragó e Délcio Basso (Orgs.)
QUESTÃO 19
Em fins do século XVIII, a Academia de Ciências da França publicou o trabalho de
C.A. de Coulomb intitulado “Primeira memória sobre a eletricidade e o magnetismo”,
no qual foram relatados a construção de uma “balança de torção” e experimentos
que relacionavam corpos carregados eletricamente com forças a distância entre
esses corpos.
Posteriormente, M. Faraday concebeu um sistema de “linhas invisíveis” que
existiriam no espaço entre as cargas elétricas, contribuindo para o desenvolvimento
do conceito de campo elétrico. Considerando esse contexto, analise as afirmações a
seguir.
I - Para Coulomb, as interações elétricas eram forças a distância entre as cargas.
II - As linhas invisíveis de Faraday não correspondem às linhas de força de um
campo elétrico.
III - O conceito de campo elétrico permitiu a substituição do conceito de ação a
distância.
Está(ão) correta(s) APENAS a(s) afirmativa(s)
(A) I
(B) II
(C) III
(D) I e II
(E) I e III.
Gabarito: E
Tipo de questão: Escolha combinada com indicação da resposta correta.
Resolutor: Prof. Me. Délcio Basso
Comentário:
A afirmativa I é correta, porque a expressão de Coulomb não leva em conta o
meio entre as cargas, considera apenas a força que cada carga exerce diretamente
sobre a outra.
ENADE Comentado 2008: Física
25
A afirmativa II é incorreta, pois as linhas invisíveis de Faraday correspondem
às linhas de força do campo elétrico.
A afirmativa III é correta uma vez que o conceito de campo elétrico substituiu
a ação a distância pela interação de cada carga com o campo.
26
Maria Eulália Pinto Tarragó e Délcio Basso (Orgs.)
QUESTÃO 20
Qual das equações do eletromagnetismo apresentadas a seguir implica a nãoexistência de monopólos magnéticos?
(A)
(B)
(C)
(D)
(E)
Gabarito: B
Tipo de questão: Escolha simples com indicação da resposta correta.
Resolutores: Elias Cantarelli Hoffmann, Profª. Dr. Maria Eulália Pinto Tarragó e
Prof. Me. Délcio Basso
Comentário:
A lei de Gauss, para o campo magnético, expressa a inseparabilidade dos
polos magnéticos e o fato de serem fechadas as linhas de indução magnética,
fazendo com que o fluxo magnético resultante através de uma superfície fechada
seja igual a zero. Em função disso, de acordo com o teorema da divergência
aplicado ao vetor indução magnética, pode-se afirmar que a equação da alternativa
(B) implica a não existência de monopolos magnéticos.
ENADE Comentado 2008: Física
27
QUESTÃO 21
Uma barra metálica é puxada de modo a deslocar-se, com velocidade , sobre dois
trilhos paralelos e condutores, separados por uma distância ℓ, como mostra a figura
abaixo.
Um resistor de resistência elétrica R conecta os dois trilhos, e um campo magnético
uniforme
atravessa, perpendicularmente, o plano do conjunto, preenchendo todo o
espaço. Qual é a intensidade da corrente elétrica que atravessa o resistor?
(A)
(B)
(C)
(D)
(E)
Gabarito: B
Tipo de questão: Escolha simples com indicação da alternativa correta.
Resolutores: Elias Cantarelli Hoffmann, Profª. Dr. Maria Eulália Pinto Tarragó e
Prof. Me. Délcio Basso
Comentário:
O fluxo magnético através do circuito está variando em função do aumento da
área correspondente aos trilhos. Em um intervalo de tempo dt , a barra metálica se
desloca em uma distância v dt e a área aumenta de dA = vdt
28
Maria Eulália Pinto Tarragó e Délcio Basso (Orgs.)
(Figura 1).
Considerando-se positivo o sentido do vetor área entrando no plano da página,

paralelamente ao vetor B , tem-se que o fluxo magnético através do circuito é
positivo e durante o intervalo de tempo dt o mesmo aumentará de acordo com:
dΦ B = BdA cos(0°) = Bvdt.
Dessa forma tem-se que a f.e.m. (força eletromotriz) induzida é dada por
ε =−
dΦ B
= − Bv .
dt
Em consequência, a corrente elétrica i no circuito será representada pelo
módulo da f.e.m. ε dividido pela resistência R, de acordo com a seguinte equação
i=
Bv
, o que corresponde a alternativa (B).
R
ENADE Comentado 2008: Física
29
QUESTÃO 22
Uma onda se propaga em uma corda, representada na figura abaixo em dois
momentos sucessivos. O intervalo de tempo entre esses dois momentos é de 0,2s.
Com relação à propagação dessa onda, foram feitas as afirmativas a seguir.
I - A velocidade da onda é 40 cm/s.
II - A freqüência da onda é 1,25 Hz.
III - As ondas estão defasadas de
.
IV - As ondas estão deslocadas de meio comprimento de onda.
São corretas APENAS as afirmações
(A) I e II
(B) I e IV
(C) II e III
(D) II e IV
(E) III e IV
Gabarito: C
Tipo de questão: Escolha combinada com indicação da resposta correta
Resolutores: Artur Majolo Scheid, Profª. Dr. Maria Eulália Pinto Tarragó e Prof. Me.
Délcio Basso
Comentário:
A figura dada na questão indica a distância entre duas cristas consecutivas,
logo se sabe que o comprimento de onda (é) λ=80,0cm. As duas ondas mostradas
30
Maria Eulália Pinto Tarragó e Délcio Basso (Orgs.)
estão deslocadas por λ/4, como indicado na figura ao lado, o que equivale dizer que
a distância percorrida pela onda foi 20,0cm no intervalo de tempo de 0,2s. Portanto,
a velocidade de propagação dessa onda é v =
20,0cm
= 100cm / s . Logo, a afirmativa I
0,2 s
é falsa.
Por outro lado, a velocidade de propagação de uma onda pode ser dada por
v = λf , isso significa que a frequência f dessa onda vale f =
v
λ
=
100cm / s
= 1,25 Hz .
80cm
Portanto, a afirmativa II é correta.
A defasagem entre as ondas é λ/4, em termos de distância percorrida pela
onda, ou
2π π
. Logo, a afirmativa III é correta.
=
2
4
A afirmativa IV é falsa, pois já comentamos que as ondas estão deslocadas
por λ/4.
Assim, a alternativa correta é a letra (C).
ENADE Comentado 2008: Física
31
QUESTÃO 23
Em uma experiência de interferência entre duas fendas iguais, utilizou-se um feixe
de luz monocromática, de comprimento de onda λ = 500 nm, incidindo
perpendicularmente ao plano que contém as fendas.
O padrão de interferência observado no anteparo, posicionado a uma distância
L=1,0 m do plano das fendas, está representado na figura a seguir com a
intensidade I em função da posição x.
Considerando-se os dados apresentados, qual é a distância d entre as duas fendas?
(A) 1,70 cm
(B) 0,85 cm
(C) 1,50 mm
(D) 0,30 mm
(E) 0,15 mm
Gabarito: D
Tipo de questão: Escolha simples com indicação da alternativa correta.
Resolutor: Profª. Dr. Elaine Evani Streck
32
Maria Eulália Pinto Tarragó e Délcio Basso (Orgs.)
Comentário:
A situação descrita na questão retrata o clássico experimento de Young que
permitiu modelar a interferência de dois feixes luminosos coerentes de comprimento
de onda λ provenientes de duas fendas separadas por uma pequena distância d
como foi mostrado na figura dada na questão.
Nessas condições, num anteparo localizado a uma distância L das fendas,
forma-se uma figura de interferência envolta por uma figura de difração, também
mostrada na questão.
As faixas claras correspondem às posições cuja interferência é construtiva,
sendo sua posição dada por dsenθ = mλ , em que m=0, 1, 2,... representa a ordem
do máximo.
A figura fornecida no enunciado permite identificar com relativa clareza que o
terceiro máximo de difração ocorre a 0,5cm do máximo central.
Levando em conta a distância de 1m entre as fendas e o anteparo, pode-se
considerar que o ângulo correspondente a esse terceiro máximo é pequeno e,
portanto, pode-se considerar que tgθ ≅ θ de modo que θ ≅
x 0 ,5cm
=
= 0 ,005rad .
L 100cm
Pelo mesmo motivo senθ ≅ θ e a relação para a posição angular dos máximos na
interferência de fenda dupla pode ser aproximada por dθ = mλ .
3 × 500 × 10−9 m
Assim sendo, tem-se para d o valor: d =
= 3 × 10− 4 m = 0,3mm .
0,005rad
Portanto, a alternativa correta é a (D).
ENADE Comentado 2008: Física
33
QUESTÃO 24
Na flauta, o tubo sonoro ressoa notas diferentes, com freqüências diferentes, de
acordo com o número de furos fechados pelos dedos do flautista.
Com os furos todos tampados, é gerada a nota lá, de 440 Hz. Abrindo alguns furos,
de modo a ressoar 2/3 do tubo, a freqüência, em hertz, será
(A) 145
(B) 293
(C) 660
(D) 880
(E) 1.000
Gabarito: C
Tipo de questão: Escolha simples com indicação da resposta correta.
Resolutores: Natthan Ruschel Soares, Profª. Dr. Maria Eulália Tarragó e Prof. Me.
Délcio Basso
Comentário:
Para resolver essa questão o estudante precisa saber que o padrão da onda
estacinária gerada dentro da flauta, com todos os furos tampados, apresenta
1
λ = L , sendo λ o comprimento de onda do som e L o comprimento da flauta.
4
Abrindo alguns furos de modo a ressoar apenas 2/3 do tubo, o comprimento de onda
ficará 2/3 menor, e, como consequência, a frequência ficará 3/2 maior. Isto é,
3
440 Hz = 660 Hz . Assim, a alternativa correta é a (C).
2
34
Maria Eulália Pinto Tarragó e Délcio Basso (Orgs.)
QUESTÃO 25
Microondas são ondas eletromagnéticas que, quando absorvidas pela água, geram
calor no interior do alimento por aumentar a vibração de suas moléculas. Na porta
de vidro de um forno de microondas existe uma rede metálica de proteção. A rede
metálica tem orifícios de 2 mm de diâmetro. Durante a operação, é possível ver o
interior do forno. No entanto, o cozinheiro está protegido da radiação microondas.
A esse respeito, foram feitas as afirmativas a seguir.
I - A radiação com comprimento de onda no infravermelho próximo (~1µm) é
bloqueada pela grade.
II - A largura dos orifícios é da ordem de grandeza do comprimento de onda da luz
visível.
III - A rede metálica impede a transmissão das microondas, mas não impede a
transmissão da radiação visível, por causa da diferença entre as freqüências.
IV - As ciências históricas têm especificidades metodológicas: seus objetos são
transitórios e atravessados por interesses de classes.
V - O comprimento de onda da radiação microondas é maior do que o da luz visível.
Está(ão) correta(s) APENAS a(s) afirmação(ões)
(A)
I
(B)
II
(C) III
(D) I e II
(E)
III e IV
Gabarito: E
Tipo de questão: Escolha combinada com indicação da resposta correta.
Resolutores: Juliane Bernardes Marcolino, Profª. Dr. Maria Eulália Tarragó e Prof.
Me. Délcio Basso
Comentário:
I – Afirmativa incorreta, pois a largura dos orifícios da grade é de 2mm, o que
é equivalente a 2000µm. Logo, radiações com comprimentos menores do que esse
valor não serão bloqueadas pela rede metálica do forno de micro-ondas.
ENADE Comentado 2008: Física
35
II – Afirmativa incorreta, já que os comprimentos de onda da luz visível estão,
aproximadamente, entre 700nm e 400nm, valores muito menores do que 2000µm.
III – Afirmativa correta, pois os comprimentos de ondas das micro-ondas
estão entre 1m e 1mm. Logo, serão barradas pela grade. Já a luz visível, de
comprimentos de ondas aproximadamente entre 0,0007mm e 0,0004mm não é
barrada pela grade.
IV - Afirmativa correta, pelo o que já foi comentado.
Portanto, a alternativa correta é a (E).
36
Maria Eulália Pinto Tarragó e Délcio Basso (Orgs.)
QUESTÃO 26
Sobre o Modelo Atômico de Böhr, são feitas as seguintes afirmações:
I - o átomo é composto de um núcleo e de uma eletrosfera;
II - o momento angular orbital do elétron é um múltiplo inteiro de h/2π, onde h é a
Constante de Planck;
III - a
freqüência
da
radiação
eletromagnética
emitida
pelo
átomo
varia
continuamente entre os dois valores correspondentes às órbitas de maior e
menor energia.
Para Böhr, é verdadeiro SOMENTE o que se afirma em
(A) I
(B) II
(C) III
(D) I e II
(E) II e III
Gabarito: D
Tipo de questão: Escolha combinada com indicação da resposta correta.
Resolutores: Profª. Dr. Elaine Evani Streck e Profª. Dr. Janaína Galho Borges
Comentário:
Quando Bohr sugeriu seu modelo para o átomo, o modelo aceito era o de
Rutherford. Para o modelo de Rutherford os elétrons giravam em torno do núcleo de
forma análoga ao movimento dos planetas em torno do Sol. Tal modelo tinha alguns
problemas não solucionados. Um deles era o fato dos elétrons não colapsarem no
núcleo, uma vez que, de acordo com a teoria eletromagnética uma carga acelerada
(aceleração centrípeta no caso) emite energia na forma de radiação o que faria com
que os mesmos descrevessem órbitas cada vez menores até colapsarem no núcleo.
Outra grande lacuna do modelo era a de que não justificava a emissão de radiação
característica (o espectro de linhas).
Bohr, para criar um modelo que solucionasse esses problemas, sugeriu que o
elétron só poderia orbitar o núcleo em órbitas circulares com energias bem definidas,
ou seja, cada órbita corresponde a um valor de energia. Quanto maior a energia do
elétron mais afastada do núcleo estaria a órbita por ele ocupada.
ENADE Comentado 2008: Física
37
Para mudar de órbita, o elétron precisava absorver, ou emitir energia. A
emissão de energia ocorre na forma de um fóton cuja energia é igual à diferença de
energia entre as órbitas.
hν = Einicial − E final
em que υ é a frequência da radiação e E final e Einicial são as energias
correspondentes às orbitas final e inicial entre as quais o elétrons realiza a transição.
O momento angular (L) , sugerido nesse modelo, é baseado nas ideias de
Planck e Einstein e é dado por: L = n
h
, em que h é a constante de Planck e n o
2π
número quântico principal e sempre assume valores inteiros e positivos (n = 1,2,3,...) .
Baseado no texto acima, os itens I e II são corretos, enquanto o item III está
errado, uma vez que a frequência da radiação eletromagnética é descontínua.
Assim, a resposta certa é a alternativa (D).
38
Maria Eulália Pinto Tarragó e Délcio Basso (Orgs.)
QUESTÃO 27
A radiação térmica emitida por estrelas pode ser modelada como semelhante à de
um corpo negro. A radiância espectral do corpo negro é máxima para uma
freqüência ou comprimento de onda. A Lei de Wien estabelece uma relação entre
esse comprimento de onda λmáx e a temperatura absoluta T do objeto, através de
uma constante determinada, experimentalmente, como igual a 2,9 x 10-3 m.K.
Usando a Lei de Wien para a estrela Polar, com λmáx= 350 nm, qual a temperatura
absoluta dessa estrela, em milhares de kelvins?
(A) 1,7
(B) 3,9
(C) 5,7
(D) 8,3
(E) 11,0
Gabarito: D
Tipo de questão: Escolha simples com indicação da alternativa correta.
Resolutor: Prof. Dr. Aldoir Rigoni
Comentário:
A energia irradiada por um corpo negro relaciona-se com a temperatura T e o
comprimento de onda λ. Quando a temperatura se eleva, a energia emitida aumenta
e o pico de distribuição desta energia se desloca para comprimentos de ondas
menores. Wien, a partir da experimentação, estabelece que o produto entre a
temperatura absoluta T e o comprimento de onda λmáx, que corresponde ao pico da
distribuição de energia, é igual a 2,9 x 10−3mK, ou seja Tλ max = 2,9 × 10 −3 mK . Usando
este modelo matemático, para o valor do comprimento de onda dado, λmáx=350nm,
determina-se que a temperatura absoluta dessa estrela, em milhares de kelvins, é
8,3; portanto a alternativa correta é a letra D.
ENADE Comentado 2008: Física
39
QUESTÃO 28
Em relação à Teoria da Relatividade Restrita, analise as afirmações a seguir.
I - O módulo da velocidade da luz no vácuo é independente das velocidades do
observador ou da fonte.
II - A Teoria Eletromagnética de Maxwell é compatível com a Teoria da Relatividade
Restrita.
III - As leis da Física são as mesmas em todos os referenciais inerciais.
Está correto o que se afirma em
(A) I, apenas.
(B) II, apenas.
(C) III, apenas.
(D) I e III, apenas.
(E) I, II e III.
Gabarito: E
Tipo de questão: Escolha combinada com indicação da resposta correta.
Resolutor: Prof. Me. Délcio Basso
Comentário:
I – Correta.
Vamos analisar a afirmativa II (“A Teoria Eletromagnética de Maxwell é
compatível com a Teoria da Relatividade Restrita”):
Os relativistas, FitzGerald, Larmor e Lorentz, alteraram o significado de
algumas grandezas nas equações do eletromagnetismo Clássico ou “de Maxwell”.


 
Em particular o significado de v na equação F = qv × B , que, originalmente, era uma
velocidade em relação ao meio onde se propagam as partículas e ondas
eletromagnéticas, chamado de éter eletromagnético (que já havia incluído o éter
luminífero) e passou a ser interpretado como velocidade em relação a um
observador inercial.
Essa alteração conceitual introduziu assimetrias no Eletromagnetismo
Clássico, tornando diversas equações não invariantes nas mudanças de referenciais
40
Maria Eulália Pinto Tarragó e Délcio Basso (Orgs.)
inerciais. Essas assimetrias foram eliminadas com o emprego das equações de
transformação de coordenadas relativísticas. Portanto, a Teoria Eletromagnética
Clássica, originalmente, não era compatível com a Teoria da Relatividade Restrita,
mas foi tornada compatível.
Na percepção do resolutor, essa afirmativa não é correta; no entanto, muitos
textos de Física não destacam o aspecto de que a Teoria Eletromagnética Clássica
foi adaptada para ficar compatível com a Teoria da Relatividade Restrita, o que
induziria a considerar essa afirmativa como correta.
III- Correta.
Assim, para o resolutor, a alternativa correta é a (D); no entanto, o gabarito dá
como alternativa correta a (E).
ENADE Comentado 2008: Física
41
QUESTÃO 29
Do ponto de vista da Física Moderna, a respeito do espectro de energias do
oscilador harmônico, são feitas as seguintes afirmações:
I - o espectro de energia é contínuo;
II - o espectro de energia é discreto;
III - em acordo com o Princípio da Correspondência de Bohr e para grandes
números qüânticos a separação de energias entre dois níveis consecutivos
torna-se desprezível quando comparada com estas energias.
Está(ão) correta(s) APENAS a(s) afirmação(ões)
(A) I
(B) II
(C) III
(D) I e II
(E) II e III
Gabarito: E
Tipo de questão: Escolha combinada com indicação da resposta correta.
Resolutores: Profª. Dr. Elaine Evani Streck e Profª. Dr. Janaína Galho Borges
Comentário:
O oscilador harmônico pode ser usado para entendermos melhor as vibrações
moleculares.
A energia potencial de um oscilador harmônico simples, a partir da equação
de Schroedinger independente do tempo, resulta em um conjunto de estados de
energia discretos dados por: E = 
1
k
 n + , para n = 0,1,2,... , em que k é a
2
m
constante da força de restauração, m a massa e  a constante de Planck dividida por
2π. Assim, dessa equação podemos inferir que o item I é errado e o II correto.
No Princípio da Correspondência, Bohr afirma que para grandes números
quânticos a mecânica quântica se reduz a mecânica clássica, criando uma relação
entre as duas teorias. O Princípio da Correspondência pode ser ilustrado, com maior
clareza, se analisarmos os níveis de energia no átomo de hidrogênio, por exemplo,
42
Maria Eulália Pinto Tarragó e Délcio Basso (Orgs.)
em que E n = −
3.6eV
. A diferença de energia entre os níveis um e dois é de 10,2eV,
n2
enquanto entre os níveis nove é dez ela é igual a 0,032eV.
Assim, a afirmação do item III é correta, pois para n muito grande já estamos
no limite da clássica.
Portanto, a resposta correta é a letra (E).
ENADE Comentado 2008: Física
43
QUESTÃO 30 - DISCURSIVA
Numa competição entre estudantes de Física de várias instituições, um grupo
projeta uma máquina térmica hipotética que opera entre somente dois reservatórios
de calor, a temperaturas de 250 K e 400 K. Nesse projeto, a máquina hipotética
produziria, por ciclo, 75 J de trabalho, absorveria 150 J de calor da fonte quente e
cederia 75 J de calor para a fonte fria.
a) Verifique se essa máquina hipotética obedece ou não à Primeira Lei da
Termodinâmica, justificando a sua resposta.
(valor: 3,0 pontos)
b) Verifique se essa máquina hipotética obedece ou não à Segunda Lei da
Termodinâmica, justificando a sua resposta.
(valor: 3,0 pontos)
c) Considerando que o menor valor de entropia é 0,1 J/K, e que o trabalho realizado
por ciclo é 75 J, esboce um diagrama “Temperatura versus Entropia” para um
Ciclo de Carnot que opere entre esses dois reservatórios de calor, indicando os
valores de temperaturas e entropias.
(valor: 4,0 pontos)
Tipo de questão: Discursiva
Resolutores: Alexandre Ferret, Maiara Oliveira Dalenogare, Márcio Galhardi, Profª.
Dr. Maria Eulália Pinto Tarragó e Prof. Me. Délcio Basso
Comentário:
a) A Primeira Lei da Termodinâmica diz que a energia é conservada segundo
a equação: ∆U = Q − W . Como a máquina recebeu 150J de calor, transforma 75J
em trabalho e perde 75J para a fonte fria, a energia se conserva, obedecendo a
primeira lei.
44
Maria Eulália Pinto Tarragó e Délcio Basso (Orgs.)
b) A Segunda Lei da Termodinâmica permite concluir que o rendimento
termodinâmico limite de um motor é e = 1 −
e = 1−
Tf
Tq
, que, nesse caso, resulta em
250 K
=37,5%. No entanto, pelo enunciado, a máquina hipotética teria um
400 K
rendimento de e =
75 J
=50,0%. Portanto, a máquina hipotética não obedece a
150 J
Segunda Lei da Termodinâmica.
c) A variação da entropia para um processo isotérmico é ∆S =
Q
T
ou
Q = ∆S .T . Assim, o trabalho no ciclo W = Qq − Q f = Tq ∆S − T f ∆S . Substituindo pelos
valores numéricos fornecidos tem-se:
75 J = 400 K∆S − 250 K∆S ; ∆S =
75 J
= 0,5 J/K.
150 K
Logo, o valor superior da entropia é (0,1+0,5)J/K=0,6J/K.
ENADE Comentado 2008: Física
45
COMPONENTE ESPECÍFICO
BACHARELADO
QUESTÃO 31
Uma dada molécula orgânica, em determinada diluição, apresenta o espectro de
absorvância descrito pela figura abaixo.
Supondo que esta molécula possa ser tratada como uma estrutura linear em que
quatro elétrons estejam aprisionados em um poço quântico infinito, qual o valor
estimado de L?
(A)
0,6 nm
(A)
1nm
(B)
0,6 µm
(C) 1 µm
(D) 2 µm
Gabarito: B
Tipo de questão: Múltipla Escolha
Resolutores: Profª. Me. Maria do Carmo Baptista Lagreca e Prof. Dr. Ricardo
Meurer Papaléo
ENADE Comentado 2008: Física
47
Comentário:
O modelo supõe quatro elétrons aprisionados num poço de potencial infinito
de largura L, em que
(1)
Pelo princípio de exclusão de Pauli, nunca pode haver mais de um elétron
ocupando o mesmo estado quântico. Como temos 4 elétrons num poço de potencial
infinito, 2 elétrons estarão no estado fundamental (n=1) e 2 elétrons estarão no
primeiro estado excitado (n=2). Considerando que os elétrons são partículas
idênticas, os elétrons têm a energia correspondente à energia do nível em que se
encontram. Assim, os 2 elétrons do nível 1 tem a mesma energia E1 e os 2 elétrons
do nível 2 tem a mesma energia E2.
Para analisarmos o gráfico da absorvância (ou absorbância), em função do
comprimento de onda, precisamos ter em mente que a absorvância, dada por
, é a capacidade intrínseca dos materiais em absorver radiações em
frequência específica. Em que I (λ ) é a intensidade da luz transmitida e I 0 (λ ) é a
intensidade da luz incidente no comprimento de onda específico λ.
O espectro de absorvância mostra claramente dois picos (um em
e
) que indicam duas transições eletrônicas permitidas.
outro em
Pela relação
, pode-se perceber que o maior comprimento de onda de
absorção, indica a transição eletrônica de menor energia: a de um elétron do estado
n=2, para o primeiro estado desocupado n=3:
(2)
Portanto, basta considerar esse primeiro pico de absorção em λ~565 nm para
determinar L a partir das equações (1) e (2). Os demais picos de absorção também
podem ser utilizados para calcular L e devem dar resultado similar. Inicialmente
vamos calcular L a partir da transição n=2 n=3.
Igualando as equações (1) e (2) para a transição considerada temos:
, usando
48
Maria Eulália Pinto Tarragó e Délcio Basso (Orgs.)
e
Assim, o valor estimado de L é 1nm, correspondente a letra (B).
ENADE Comentado 2008: Física
49
QUESTÃO 32
Num dia de chuva, uma nuvem eletricamente carregada pode se descarregar
produzindo relâmpagos. Uma nuvem típica se encontra a uma altura de 5.000 m do
solo, com uma diferença de potencial de 10 milhões de volts em relação ao solo. Em
um laboratório, uma estudante de Física realiza uma experiência para medir a
rigidez dielétrica do ar seco usando um capacitor de placas planas e paralelas cuja
distância entre as placas pode ser variada. Mantendo uma diferença de potencial
constante entre as placas e iguais a 24 kV, a estudante diminui lentamente a
distância entre elas até que, na distância de 0,8 cm, observa uma centelha no ar
entre as placas.
Quais são os valores do campo elétrico entre a nuvem e o solo e da rigidez dielétrica
do ar seco, respectivamente?
(A)
2,0 kV/m e 3,0 x 106 V/m
(B)
2,0 kV/m e 1,9 x 104 V/m
(C) 5,0 kV/m e 3,0 x 106 V/m
(D) 10 kV/m e 3,0 x 105 V/m
(E)
20 kV/m e 1,9 x 106 V/m
Gabarito: A
Tipo de questão: Escolha simples com indicação da resposta correta.
Resolutores: Elias Cantarelli Hoffmann, Profª. Dr. Maria Eulália Pinto Tarragó e
Prof. Me. Délcio Basso
Comentário:
Para se encontrar o valor da rigidez dielétrica do ar seco, deve-se calcular o
valor do módulo do campo elétrico máximo, entre as placas planas do capacitor,
para que não ocorra ruptura dielétrica. Considerando-se que a diferença de potencial
entre as placas do capacitor é constante, e que na distância de 0,80 cm entre as
placas tem-se a intensidade máxima do campo elétrico, antes da ruptura, o valor da
rigidez dielétrica é:

∆V placas
24 kV
24 × 103 V
=
=
= 3,0 × 106V / m .
Emáx =
∆x placas 0,80 cm 8,0 × 10− 3 m
50
Maria Eulália Pinto Tarragó e Délcio Basso (Orgs.)
Semelhantemente, o valor do campo elétrico entre a nuvem e o solo é:
nuvem
 ∆Vsolo
10 × 106 V
E =
=
= 2,0 × 103 V/m=2,0kV/m.
nuvem
∆xsolo
5000m
Assim, é correta a alternativa (A).
ENADE Comentado 2008: Física
51
QUESTÃO 33
Quando uma onda eletromagnética plana penetra em um meio material, a sua
amplitude decai com a distância de penetração, ou seja, ela tem a sua amplitude
atenuada pelo meio. A profundidade de penetração da onda δ é a profundidade na
qual a intensidade do campo foi reduzida a aproximadamente 1/3 do valor inicial.
Define-se a profundidade da penetração como:
onde
σ é a condutividade do meio;
µ é a permeabilidade magnética do meio;
ω é a freqüência angular da onda.
Para onda com freqüência específica ω0, a condutividade na prata é σprata = 3x10-7
(mΩ)-1 e no mar é σmar= 4,0 x 10-7 (mΩ)-1, e para ambos é µ= 4π x 10-7 N/A2.
A esse respeito, analise as afirmações a seguir.
I - A penetração da onda é maior na prata do que no mar.
II - Para um meio condutor com condutividade constante, uma onda com menor
comprimento de onda tem uma profundidade de penetração maior do que outra
onda com maior comprimento de onda.
III - A uma profundidade de 2δ da superfície, a sua amplitude será aproximadamente
10% da amplitude original.
Estão corretas SOMENTE as afirmações
(A)
I
(B)
II
(C) III
(D) I e III
(E)
II e III
Gabarito: D
Tipo de questão: Escolha combinada com indicação da resposta correta.
Resolutores: Profª. Dr. Maria Eulália Pinto Tarragó e Prof. Me. Délcio Basso
52
Maria Eulália Pinto Tarragó e Délcio Basso (Orgs.)
Comentário:
Essa é uma questão de interpretação de texto, pois todas as informações
estão dadas no enunciado. Portanto, passaremos diretamente a análise das
afirmativas.
A profundidade de penetração da onda δ é maior na prata do que no mar, pois
δ é inversamente proporcional à raiz da condutividade elétrica σ, e σprata<σmar;
portanto, a afirmativa I está CORRETA.
A Velocidade angular ω pode ser dada por ω = 2πf = 2π
v
λ
, sendo λ o
comprimento de onda. Então, para um meio condutor com condutividade constante a
profundidade de penetração da onda δ é diretamente proporcional à raiz de λ;
portanto, a afirmativa II é INCORRETA.
A profundidade de penetração da onda δ é a profundidade na qual a
intensidade do campo, ou seja, a amplitude da onda, é 1/3 do valor inicial. Logo,
quando a profundidade de penetração for 2δ a amplitude será
1 1 1
× = ≅ 0,10 ;
3 3 9
portanto, a afirmativa III está CORRETA.
Assim, a letra correta é a (D).
ENADE Comentado 2008: Física
53
QUESTÃO 34 (QUESTÃO ANULADA)
O enxofre cristalino, um sólido transparente de cor amarelo pálido por absorver
apenas a luz azul e nenhuma outra cor, é um isolante elétrico. Qual é o gap de
energia do enxofre cristalino?
(Considere hc = 1,2 eV.µm)
(A)
2,5
(B)
4,5
(C) 5,0
(D) 5,5
(E)
6,0
Gabarito: A
Tipo de questão: Escolha simples com indicação da resposta correta.
Resolutores: Prof. Dr. Adriano Moehlecke, Profª. Dr. Izete Zanesco e Profª. Dr.
Aline Cristiane Pan
Comentário:
Um cristal só pode absorver os fótons de luz que apresentam energia
equivalente a seu gap de energia. Por outro lado, sabe-se que a energia de um
fóton de luz é dada por E =
hc
λ
, em que h é a constante de Planck, c é a velocidade
da luz no vácuo e λ é o comprimento de onda da luz. Portanto, para resolver essa
questão devemos lembrar que o comprimento de onda correspondente ao azul é
0,48µm e, consequentemente, E =
54
1,2eV µm
= 2,5eV. No entanto, caso o aluno não
0,48µm
Maria Eulália Pinto Tarragó e Délcio Basso (Orgs.)
se lembre do λ correspondente ao azul, mesmo que utilizasse qualquer outro λ
na faixa do espectro visível, encontraria que a alternativa mais próxima do valor
que obteve é a (A).
Logo, a resposta certa é a letra (A). Observa-se que na questão fornecida,
faltou explicitar a unidade de energia para as alternativas a serem escolhidas.
ENADE Comentado 2008: Física
55
QUESTÃO 35
Um metal unidimensional tem um elétron de condução por átomo a temperatura
T=0K. O espaçamento interatômico no metal é D. Supondo que os elétrons movemse livremente, qual é a energia de Fermi EF?
( = h / 2π é a Constante de Planck e m é a massa do elétron)
(A)
(B)
(C)
(D)
(E)
Gabarito: A
Tipo de questão: Múltipla Escolha
Resolutor: Prof. Dr. Cássio Stein Moura
Comentário:
Nesse problema, podemos aproximar cada átomo do cristal como um poço de
potencial quadrado infinito. Por ser um metal, consideramos que o elétron de
condução move-se livremente no interior do poço de potencial e, portanto, a
equação de Schrödinger unidimensional independente do tempo para o problema é:
 2 ∂ 2ψ
−
= Eψ .
2m ∂x 2
56
Maria Eulália Pinto Tarragó e Délcio Basso (Orgs.)
ψ ( x = 0) = 0
Considerando-se as condições de contorno: 
, esta equação
ψ ( x = D) = 0
diferencial apresenta como solução a autofunção: ψ ( x) = A sin(kx) , em que k é
definido pelo autovalor de energia E =
2k 2
.
2m
Para que as condições de contorno sejam obedecidas, ou seja, a função de
onda se anule nas paredes é necessário que: k =
 2π 2 2
nπ
. Logo, En =
n ,
D
2mD 2
n
=
1, 2, 3 ...
Pelo princípio de exclusão de Pauli, em se tratando de férmions, cada nível
energético (n =1, 2, 3 ... ) somente pode ser ocupado por no máximo dois elétrons.
Se considerarmos que o metal está sujeito a temperatura nula e que hajam N
elétrons no poço de potencial, todos os níveis até n =
N
estão ocupados.
2
Defini-se como energia de Fermi (EF) a energia do último nível ocupado, ou
N
 2π 2  N 
seja, o nível em que n = . Assim, E F = E n= N / 2 =
  . Como, do enunciado,
2
2mD 2  2 
2
 2π 2  1 
 2π 2
.
N= 1, chegamos à expressão para a energia de Fermi: E F =
=


2mD 2  2 
8mD 2
2
Assim, a alternativa correta é a (A).
ENADE Comentado 2008: Física
57
QUESTÃO 36
O LHC (Large Hadron Collider), acelerador de partículas que entrou em operação
este ano, busca uma nova Física na escala de até 14 TeV. A principal busca é pela
partícula chamada Higgs, que supostamente gera as massas das partículas
responsáveis pela interação nuclear fraca, como o W + e o W −. Essas partículas são
muito massivas se comparadas a outras como o próton e o elétron. Suas massas de
repouso são da ordem de 82 GeV. Elas serão geradas em quantidade no LHC e
com energias que podem chegar, em um experimento típico, a 500 GeV para o W +
ou o W −. Essas partículas são muito instáveis, pois decaem rapidamente. Estima-se
que suas vidas médias sejam de 3 x 10−25 s, em seu referencial de repouso. No
referencial do laboratório (LHC), qual seria sua vida média, num experimento típico?
(Dados: 1 TeV = 103 GeV = 1012eV
1eV = 1,6 x 10−19J)
(A)
9 x 10−25s
(B)
18 x 10−25s
(C) 27 x 10−25s
(D)
3 x 10−24s
(E)
18 x 10−24s
Gabarito: B
Tipo de questão: Múltipla Escolha
Resolutores: Prof. Me. Délcio Basso e Profª. Dr. Maria Eulália Pinto Tarragó
Comentário:
De acordo com a Teoria da Relatividade Restrita (TRR), defini-se tempo
próprio ∆t0 como o intervalo de tempo medido no próprio referencial no qual o
evento ocorre. Se um observador estiver em outro referencial, deslocando-se com
velocidade constante v em relação ao referencial próprio, o intervalo de tempo
∆t que ele irá obter para o evento será dado por ∆t =
∆t o
v2
1− 2
c
ou ∆t = γ ∆t o , sendo γ
o fator de Lorentz. Como a velocidade v é sempre menor do que c, então γ é sempre
maior do que a unidade (ou igual, para os casos em que v<<c). Isso significa que o
58
Maria Eulália Pinto Tarragó e Délcio Basso (Orgs.)
intervalo de tempo medido por qualquer observador que esteja em outro referencial
inercial (que não seja o próprio) será sempre maior do que o tempo próprio (ou igual,
para os casos em que v<<c); por isso, esse fenômeno chama-se “dilatação do
tempo”.
Quanto à energia, a TRR nos diz que a energia total E de uma partícula com
energia cinética K é dada por E = E 0 + K , sendo E 0 = m0 c 2 a energia associada à
massa de repouso m0. A energia total E também pode ser dada por E = γ E0 .
No LHC, a razão entre a energia total e a energia de repouso para a partícula
de Higgs, caso ela venha a ser detectada, seria
E 500GeV
=
= γ = 6,1 . Isso
E0
82GeV
significaria que a vida média dessas partículas, medida no referencial do laboratório,
será
seis
vezes
a
vida
média
medida
no
referencial
próprio,
isto
é: ∆t = γ ∆t o = 6,1 × 3,0 × 10−25 s = 18 × 10−25 s .
Assim, a alternativa correta é a (B).
ENADE Comentado 2008: Física
59
QUESTÃO 37
O urânio natural presente na Terra é uma mistura de
vida média do
238
U é 4,5 bilhões de anos e a do
238
U (99,3%) e
235
U (0,7%). A
235
U é 1,0 bilhão de anos. Supondo
que, na explosão de uma supernova, esses isótopos tenham sido produzidos em
quantidades iguais, há quanto tempo, em anos, deve ter ocorrido essa explosão?
(Considere ln(99,3/0,7) = 5)
(A)
6 mil
(B)
20 milhões
(C) 1 bilhão
(D) 6 bilhões
(E)
15 bilhões
Gabarito: D
Tipo de questão: Múltipla Escolha
Resolutores: Profª. Dr. Maria Eulália Pinto Tarragó e Prof. Me. Délcio Basso
Comentário:
Para resolver essa questão o estudante deve lembrar que as transformações
radiativas seguem a lei N1 = N10 e
− t
τ1
, na quais N1 é o número de núcleos radiativos
do tipo 1 presentes na amostra no tempo t, N10 é número de núcleos radiativos do
tipo 1 presentes na amostra no tempo inicial t=0 e τ1 é a vida-média dos núcleos do
tipo 1, ou seja, o tempo que em média um núcleo permanece sem desintegrar.
De acordo com as informações da questão, podemos escrever que o número
de núcleos de U238 em função do tempo é N1 = N10 e
em função do tempo é N 2 = N 20 e
− t
-se
− t
τ2
− t
τ1
(Eq.2). Dividindo-se a (Eq.1) pela (Eq.2) obtém-
− t
( 1 − 1 )t
N1 N10 e τ 1
e τ1
τ2
τ1
=
, pois foi dito que esses núcleos foram
=
=
e
− t
− t
τ2
N2 N 0 e τ 2
e
2
produzidos em quantidades iguais ( N10 = N 20 ). Portanto, ln
60
(Eq.1) e o número de U235
Maria Eulália Pinto Tarragó e Délcio Basso (Orgs.)
N1
1 1
= ( − )t .
τ 2 τ1
N2
Inserindo
os
valores
numéricos
fornecidos
obtemos:
N1
5
N2
t=
=
≈ 6 × 109 anos .
1 1
1
1
−
( − ) (
)
τ 2 τ1
1 × 109 4,5 × 109
ln
Assim, a alternativa correta é a (D).
ENADE Comentado 2008: Física
61
QUESTÃO 38
O céu é azul devido ao espalhamento da luz solar pelas moléculas da atmosfera
distribuídas de forma inomogênea. Este espalhamento, denominado espalhamento
Rayleigh, também importante em propagação de luz em fibras ópticas, varia com o
inverso da quarta potência do comprimento de onda (1/λ4).
Considerando
essas
informações,
analise
as
explicações
dos
fenômenos
apresentados a seguir.
I - Em propagação de luz em fibras ópticas de vidro, o Espalhamento Rayleigh é
responsável por uma atenuação maior da intensidade na transmissão óptica
para comprimentos de onda da luz visível do que para a radiação infravermelha.
II - A cor avermelhada do pôr do sol ocorre porque, ao entardecer, os raios solares
incidem tangencialmente à superfície da Terra e as cores de maior freqüência
não conseguem atravessar toda a extensão da atmosfera.
III - A cor azul do céu ocorre porque a luz solar, ao passar pela atmosfera, sofre um
espalhamento maior para as radiações de menor comprimento de onda do que
para as de maior comprimento de onda.
Está(ão) correta(s) a(s) explicação(ões)
(A)
I, apenas.
(B)
I e II, apenas.
(C) I e III, apenas.
(D) II e III, apenas.
(E)
I, II e III.
Gabarito: E
Tipo de questão: Múltipla Escolha
Resolutor: Prof. Dr. Cássio Stein Moura
Comentário:
Vamos discutir cada uma das explicações propostas na questão:
I – Conforme o enunciado afirma, o espalhamento Rayleigh é proporcional a
1
λ4
. A luz visível tem um comprimento de onda na faixa que vai de
aproximadamente 400 a 780 nm, enquanto que o infravermelho situa-se entre o
62
Maria Eulália Pinto Tarragó e Délcio Basso (Orgs.)
limite de maior comprimento de onda do visível, 780 nm, e cerca de 1 mm para o
infravermelho longínquo. Como o comprimento de onda do infravermelho é maior
que o do visível, o fator 1
λ4
será pequeno para o infravermelho em comparação
com a porção visível do espectro eletromagnético. Sendo assim, podemos afirmar
que a luz visível sofre uma atenuação maior que o infravermelho ao se propagar
através de fibras óticas.
II – Se considerarmos o planeta Terra e a sua atmosfera como esferas
concêntricas, podemos afirmar, do ponto de vista geométrico, que a camada de ar
percorrida por um raio de luz é maior quando ele atinge tangencialmente a superfície
do que quando a atinge perpendicularmente. Dessa forma, o efeito de espalhamento
Rayleigh será mais evidente no caso da incidência tangencial, que é o caso do pôr
do Sol, do que ao meio-dia quando a luz atinge a superfície perpendicularmente. Os
tons vermelhos do espectro visível têm comprimentos de onda maiores (ou
frequências menores) do que os tons azuis, o que faz com que estes sofram um
espalhamento mais intenso do que aqueles, devido ao fator 1
λ4
, de forma similar à
explicação I acima. Devido ao maior espalhamento dos raios azuis, os tons
alaranjados e vermelhos predominam, dando as cores características do pôr do Sol.
III – Se o planeta Terra não possuísse atmosfera, a luz não sofreria
espalhamento Rayleigh e o céu pareceria negro, como é o caso da Lua. Vemos o
céu terrestre da cor azul, porque a luz ao passar pela atmosfera sofre espalhamento:
os tons vermelhos tendem a espalhar menos e a chegar de forma direta na
superfície, enquanto que os tons azuis são espalhados. A cor que nossos olhos
identificam é justamente dos feixes espalhados na atmosfera, dando-lhe a cor
característica azul.
Assim, a alternativa correta é a (E).
ENADE Comentado 2008: Física
63
QUESTÃO 39 – DISCURSIVA
Uma partícula de massa m desliza sem atrito em um anel de raio R. O anel gira com
velocidade angular constante em torno de um eixo vertical, como mostra a figura
acima. A aceleração da gravidade é g.
a) Encontre a lagrangiana do sistema, usando como coordenada generalizada o
ângulo definido na figura.
(valor: 3,0 pontos)
b) Escreva a Equação de Euler-Lagrange desse sistema.
(valor: 3,0 pontos)
c) Quantos pontos de equilíbrio (estáveis ou instáveis) existem para Ω² < g/R e para
Ω² > g/R ?
(valor: 4,0 pontos)
Tipo de questão: Discursiva
Resolutor: Profª. Dr. Sayonara Salvador Cabral da Costa
Comentário:
a) A lagrangiana L do sistema conservativo da questão é a diferença entre a
energia cinética T da partícula e a energia potencial gravitacional U a ela associada,
ou seja, L = T – U. Usando como coordenada generalizada θ, passamos a escrever
estas duas energias:
A energia cinética T da partícula é dada pela soma de duas parcelas: a
primeira, T1, resultante do movimento da partícula deslizando sobre o anel
64
Maria Eulália Pinto Tarragó e Délcio Basso (Orgs.)
( T1 =
1
mR 2θ 2 ) e a segunda, T2, resultante do movimento do anel em torno do eixo
2
vertical ( T2 =
1
m(Rsenθ ) 2 Ω 2 );
2
Para a energia potencial gravitacional, U=mgh, pode-se escolher como
referencial U=0 para θ=0 e h=0. Dessa forma, U=mgR(1−cosθ).
1
1
Portanto, L = mR 2θ 2 + m(Rsenθ ) 2 Ω 2 + mgRcosθ − mgR .
2
2
b) A equação de Euler-Lagrange,
∂L d  ∂L 
−   = 0 , para o sistema resulta,
∂θ dt  ∂θ 
então:
mR 2 senθ . cos θ .Ω 2 + mgR(-senθ ) - mR 2θ = 0 ,
que pode ser escrita, convenientemente, como:


θ =  Ω 2 cosθ −
g
 senθ .
R
c) Os pontos ou posições de equilíbrio serão aqueles para os quais vale a
condição de θ = 0 .
Da equação de Euler-Lagrange, obtida acima, vê-se que essa condição
implica: senθ = 0 ou cosθ =
g
.
RΩ 2
No primeiro caso, as posições de equilíbrio são θ=0 e θ=π.
No segundo caso, se Ω2>g/R, sendo cosθ=g/RΩ2, resulta que θ=cos-1(g/RΩ2);
Então, se Ω2<g/R, resulta que cosθ>1, o que é impossível, logo as posições
de equilíbrio resultam as do primeiro caso, ou seja, θ=0 (equilíbrio estável) e θ=π
(equilíbrio instável).
ENADE Comentado 2008: Física
65
QUESTÃO 40 - DISCURSIVA
A figura abaixo mostra o espectro de absorção de vibração-rotação de uma molécula
diatômica heteronuclear na temperatura ambiente. Para moléculas desse tipo, as
energias vibracionais-rotacionais são dadas por
onde
j é o número quântico rotacional e n é o número quântico vibracional;
I é o momento de inércia da molécula e ν0 é a freqüência de vibração clássica da
molécula. As transições mostradas correspondem às transições com nfinal=1 e
ninicial=0 (∆n = 1) e (∆j = ± 1).
(Dados: h = 4,0 x 10
−15
eVs; π = 10; massa reduzida da molécula µ= 1,5 x 10
2
−27
2
kg; (0,360) = 0,13)
a) Apresente a expressão da energia das radiações absorvidas em função do
momento de inércia.
(valor: 4,0 pontos)
b) Calcule o valor numérico aproximado do momento de inércia da molécula.
(valor: 3,0 pontos)
c) Calcule o valor numérico aproximado da constante elástica da molécula.
(valor: 3,0 pontos)
66
Maria Eulália Pinto Tarragó e Délcio Basso (Orgs.)
Tipo de questão: Discursiva
Resolutor: Prof. Dr. Cássio Stein Moura
Comentário:
a) Como a molécula está na temperatura ambiente, podemos considerar que
as energias dos estados vibratórios são suficientemente elevadas em comparação
com a energia térmica kT e, portanto, a molécula se encontra no estado vibracional
fundamental no qual n = 0 .
Nessa situação, as regras de seleção permitem que, para a transição
ninicial = 0 para n final = 1 , o número quântico rotacional sofra uma variação ∆j = ± 1, ou
seja, j final = jinicial + 1 ou jinicial = j final − 1 .
A energia dos autoestados vibro-rotacionais é dada, conforme o enunciado da
questão, por:
h2
1

E (n, j ) =  n + hν0 + 2 j ( j +1)
2
8π l

[1]
Da equação [1] podemos escrever a energia absorvida para cada uma das
transições permitidas:


h2
h2
1
 1
E (0 → 1, j → j + 1) = E (1, j + 1) − E (0, j ) = 1 + hν0 + 2 ( j + 1)( j + 1 + 1) −  0 + hν0 + 2 j ( j + 1)
2
8π l
8π l
 2


E (0 → 1, j → j + 1) = hν0 +
h2
( j +1)
4π 2 l
[2]


h2
h2
1
 1
E (0 → 1, j → j − 1) = E (1, j − 1) − E (0, j ) = 1+ hν0 + 2 ( j − 1)( j − 1+ 1) −  0 + hν0 + 2 j ( j + 1)
2
8π l
8π l
 2


E (0 → 1, j → j − 1) = hν0 −
h2
j
4π 2 l
[3]
ENADE Comentado 2008: Física
67
b) Analisando o espectro apresentado no enunciado da questão, podemos
extrair os valores das energias de transições a seguir:
∆E(0 → 1, 0 → 1) = 0,363 eV
[4]
∆E(0 → 1, 1 → 0) = 0,358 eV
[5]
Substituindo [4] e [5] e os respectivos valores de n e j nas equações [2] e [3],
ficamos com:
0,363[eV ] = 4,0 × 10
−15
0,358[eV ] = 4,0 × 10
−15
[eV ⋅ s ]ν + (4,0 × 10 [eV ⋅ s ]) ⋅ (0 +1)
4 ⋅ 10 ⋅ l
[6]
[eV ⋅ s ]ν − (4,0 × 10 [eV ⋅ s ]) ⋅ (1)
4 ⋅ 10 ⋅ l
[7]
2
−15
0
2
−15
0
Resolvendo [6] e [7] para l encontramos:
l = 1,6 × 10 −28 eV ⋅ s 2
[8]
c) Para os níveis vibracionais mais baixos, a vibração da molécula diatômica
pode ser comparada a um oscilador harmônico simples, cuja frequência de oscilação
é dada pela relação entre a constante elástica k e a massa efetiva µ:
ν0 =
1
2π
k
μ
[9]
Para calcular a constante elástica do oscilador usando a equação [9],
precisamos conhecer o valor de sua frequência fundamental. Para encontrá-la,
substituímos o valor de l definido em [8] na equação [6] (ou na [7]):
0,363[eV ] = 4,0 × 10
−15
[eV ⋅ s ]ν + (4,0 × 10 [eV ⋅ s ])
4 ⋅ 10 ⋅ 1,6 × 10 [eV ⋅ s ]
0
Com isso, chegamos a
68
2
−15
Maria Eulália Pinto Tarragó e Délcio Basso (Orgs.)
− 28
2
[10]
ν0 = 9,0 × 1013 Hz
[11]
Substituindo [11] em [9],
9,0 × 1013 =
1
k
2π 1,5 × 10 − 27
[12]
e, resolvendo para k, temos que:
k = 4,8 × 10 2 N / m .
ENADE Comentado 2008: Física
69
COMPONENTE ESPECÍFICO
LICENCIATURA
QUESTÃO 41
Para avaliar se os estudantes haviam superado concepções comuns às da teoria
medieval do impetus em relação à compreensão dinâmica da situação estudada, o
professor propôs o problema apresentado a seguir.
Uma bola de futebol é lançada verticalmente para cima, a partir do telhado de
um edifício de altura h0, com velocidade v0. Apresente uma explicação relativa
ao lançamento, que leve em conta a resistência do ar.
Qual das seguintes seria a resposta típica de um aluno dito “newtoniano”?
(A)
A força com que a bola foi lançada diminui com o tempo, até se igualar, na
posição de altura máxima, à soma das forças peso e atrito com o ar.
(B)
A força com que a bola foi lançada diminui pela ação do atrito com o ar, até se
igualar ao peso da bola na posição de altura máxima.
(C) As forças que agem sobre a bola após o lançamento agem no sentido contrário
ao movimento na subida, e a favor do movimento, na descida.
(D) As forças que agem sobre a bola após o lançamento agem no sentido contrário
ao movimento na subida, e em ambos os sentidos, na descida.
(E)
As forças que agem sobre a bola após o lançamento agem no mesmo sentido
que o movimento na subida e na descida.
Gabarito: D
Tipo de questão: Múltipla Escolha
Resolutor: Prof. Dr. João Bernardes da Rocha Filho
Comentário:
Essa é uma questão que tenta avaliar se a noção de força versus movimento
do estudante universitário é aristotélica ou newtoniana, e se ele compreende
adequadamente as ideias de atrito e peso.
A questão pode ser resolvida pensando-se que o movimento de uma bola
lançada nessas condições pode ser subdividido em três momentos distintos, para
efeitos de análise:
1) o instante do lançamento, isto é, o curto intervalo de tempo no qual o pé do
lançador, ou outro mecanismo de lançamento, encosta na bola;
ENADE Comentado 2008: Física
71
2) o período que vai do instante em que o lançador desencosta da bola até
que ela atinja a altura máxima, e;
3) o período que corresponde a descida da bola.
Sobre esses três momentos podemos afirmar que:
Tanto o lançador quanto a bola são entidades em certa medida elásticas, a
força com que a bola é lançada varia ao longo do intervalo de tempo em que o
lançador fica em contato com ela, mas cessa imediatamente quando termina o
contato da bola com o lançador, pois é justamente esse contato o meio através do
qual a força é aplicada. Não havendo contato, não há força de lançamento, por isso
ela termina quando a bola se liberta do contato com o lançador.
As forças que agem sobre a bola após o lançamento são (basicamente) duas:
o atrito com o ar e o peso da bola. O peso age puxando a bola para baixo durante
toda a trajetória (subida e descida), já que depende apenas da ação gravitacional, e
não do movimento da bola. O atrito, por sua vez, é uma força que se opõe ao
movimento do objeto, portanto, o atrito da bola com o ar é uma força que aponta
para baixo, durante sua subida e, para cima, durante sua descida.
Com base nas considerações acima, podemos agora avaliar cada uma das
alternativas oferecidas na questão:
(A) está errada, pois a força com que a bola foi lançada se reduz a zero
imediatamente na separação da bola e do lançador;
(B) está errada, pela mesma razão do item (A);
(C) está errada, pois as forças que atuam na descida da bola são o peso, que
atua a favor do movimento, e o atrito, que atua contra o movimento;
(D) está correta, pois na subida a força peso e a força de atrito com o ar se
opõem ao movimento, mas na descida o peso favorece o movimento
enquanto o atrito se opõe a ele, apontando em sentidos opostos;
(E) está errada, pois na subida o atrito e o peso se opõem ao movimento, e
na descida o peso age no mesmo sentido do movimento, enquanto o
atrito se opõe a este.
72
Maria Eulália Pinto Tarragó e Délcio Basso (Orgs.)
QUESTÃO 42
Hertz, no experimento em que evidenciou a existência das ondas eletromagnéticas,
notou que a descarga elétrica no sensor era mais facilmente percebida quando este
era iluminado com luz de freqüência acima de um certo valor. A explicação de
Einstein para este efeito, denominado fotoelétrico, considera que
(A)
o aumento da intensidade da luz implica um aumento do número de fótons de
mesma energia que incide sobre o sensor.
(B)
o intervalo de tempo entre a chegada da luz ao sensor e a emissão dos
elétrons é diferente de zero.
(C) a luz se comporta como onda no momento em que ocorre o efeito.
(D) a energia dos elétrons que saem do sensor depende diretamente da
intensidade de luz incidente.
(E)
a energia do fóton incidente é igual à energia cinética do elétron atingido.
Gabarito: A
Tipo de questão: Múltipla Escolha
Resolutor: Prof. Dr. João Bernardes da Rocha Filho
Comentário:
O objetivo dessa questão é avaliar se o estudante compreende o efeito
fotoelétrico.
O texto da questão alude a um experimento realizado por Hertz, e se refere
explicitamente à relação entre a frequência da radiação luminosa emitida e a
ocorrência perceptível de descargas elétricas entre sensores, associando esse
fenômeno ao efeito fotoelétrico estudado por Einstein. O cerne do efeito fotoelétrico
envolve justamente o compromisso entre a frequência da radiação emitida e a
capacidade desta em produzir liberações de elétrons em superfícies metálicas.
Para responder à questão o estudante deveria saber que:
1)
a intensidade de uma fonte luminosa está relacionada à quantidade de
fótons que ela emite em certo tempo, ou seja, a taxa de emissão de
fótons;
2)
o efeito fotoelétrico é um fenômeno que sugere que a radiação
luminosa, normalmente considerada como onda eletromagnética, tem
também comportamento de partícula;
ENADE Comentado 2008: Física
73
3)
a energia com que os elétrons são liberados de uma superfície metálica
iluminada é função da energia da radiação incidente e da função
trabalho do metal;
4)
a intensidade da radiação incidente tem relação com a quantidade de
elétrons liberados em certo intervalo de tempo, ou seja, que a taxa de
fótons incidentes tem relação com a taxa de liberação de elétrons;
5)
parte da energia do fóton incidente é absorvida para garantir a liberação
do elétron, e que a energia excedente do fóton é transformada em
energia cinética do elétron;
6)
o efeito fotoelétrico é um fenômeno quântico para o qual não está
definido um intervalo de tempo e, portanto, podemos dizer que o elétron
é liberado simultaneamente à incidência do fóton.
Com base nas informações acima, podemos agora avaliar cada uma das
alternativas oferecidas nessa questão:
(A) está correta, pois o aumento na intensidade da luz é precisamente o
aumento na taxa com que os fótons são liberados pela fonte luminosa e,
portanto, na taxa com que atingem o sensor;
(B) está errada, pois considera-se que o efeito fotoelétrico, por ser quântico,
não tem tempo definido para sua ocorrência, sendo instantâneo;
(C) está errada, pois o efeito fotoelétrico é justamente um fenômeno que
envolve a expulsão de elétrons superficiais de metais, o que sugere que
os fótons têm momento linear e, portanto, comportam-se como partículas;
(D) está errada, pois a quantidade dos elétrons liberados pela radiação está
relacionada com a intensidade da radiação luminosa incidente, enquanto
a energia desses elétrons tem relação com a frequência da luz incidente e
com a função trabalho do metal;
(E) está errada, pois uma parte da energia do fóton incidente é utilizada para
extrair o elétron de sua órbita atômica, e somente a parte restante é
transformada em energia cinética.
74
Maria Eulália Pinto Tarragó e Délcio Basso (Orgs.)
QUESTÃO 43
Há uma variedade de possibilidades e tendências do uso de estratégias de ensino
frutíferas para se ensinar de modo significativo e consistente. Uma abordagem
construtivista a ser adotada no laboratório didático é a que apresenta situações
experimentais destinadas a que os alunos
(A)
verifiquem e confirmem leis e teorias da Física, previamente ensinadas.
(B)
revelem qualitativamente suas idéias prévias.
(C) exemplifiquem o uso da metodologia científica na produção da ciência.
(D) redescubram a ciência produzida por cientistas.
(E)
evitem o desenvolvimento de concepções alternativas à científica.
Gabarito: B
Tipo de questão: Múltipla Escolha
Resolutor: Prof. Dr. João Bernardes da Rocha Filho
Comentário:
O objetivo da questão é avaliar o conhecimento que os estudantes têm acerca
do construtivismo, e para isso apresenta uma situação hipotética envolvendo um
experimento proposto por um professor em um laboratório didático, procurando
identificar, entre as alternativas, a que representa melhor uma abordagem
construtivista adequada para a situação experimental.
Para responder essa questão o estudante deveria saber que o construtivismo
é uma epistemologia com aplicação metodológica e didática no ensino das ciências,
que considera que o conhecimento não é algo pronto, a ser adquirido, assimilado ou
transmitido, mas sim elaborado internamente em cada indivíduo pela interação que
realiza com o meio, com a produção científica da humanidade, com a cultura popular
e com as demais pessoas. Nesse sentido, o conhecimento é produto da ação, que
por sua vez é fundada nos conhecimentos prévios que todos têm, e que, portanto,
precisam ser considerados no processo educativo.
Com base nas informações acima, podemos agora avaliar cada uma das
alternativas oferecidas na questão:
ENADE Comentado 2008: Física
75
(A) está errada, pois a verificação e a confirmação de leis e teorias da Física,
previamente ensinadas, caracteriza um ensino tradicional, centrado no
conhecimento como algo externo, pronto, dominado pelo professor, a ser
assimilado pelo educando;
(B) está correta, pois um dos objetivos do uso do laboratório ou da
experimentação no contexto construtivista é fazer aflorar os préconhecimentos dos estudantes, permitindo que eles os manifestem por
meio da manipulação da realidade. A partir desses pré-conhecimentos e
de conhecimentos obtidos de todas as outras fontes acessíveis, é, então,
possível que o estudante empenhe-se em um processo de reconstrução
própria do conhecimento científico;
(C) está errada, pois a metodologia científica é baseada no pressuposto de
que o conhecimento pode ser atingido pela utilização de métodos formais,
hipotético-dedutivos e lineares, que o construtivismo não reconhece como
úteis no contexto didático de um laboratório de Física, pois eles exigem
pré-conhecimentos homogêneos, incluindo metodologias padrão, que não
se originam no próprio indivíduo;
(D) está errada, pois a redescoberta da ciência produzida por cientistas
pressupõe que o caminho que leva ao conhecimento educacional é o
mesmo que leva às descobertas científicas, como relatadas nos livros de
História da Ciência. Para o construtivismo, além da repetição ser
naturalmente contraproducente em termos de aprendizagem, o caminho
seguido pelos cientistas para a descoberta de leis é frequentemente
intuitivo e serendípico, e dificilmente pode ser reproduzido em laboratórios
didáticos em função do tempo, da dedicação extrema que demandam, e
dos condicionantes históricos diferentes;
(E) está errada, pois as concepções alternativas existem naturalmente nos
estudantes, e a ida ao laboratório de Física, no contexto construtivista,
objetiva exatamente que essas concepções aflorem e sejam questionadas
pelos próprios estudantes, a partir dos resultados que encontram na
manipulação da realidade e nas demais fontes de consulta que puderem
acessar.
76
Maria Eulália Pinto Tarragó e Délcio Basso (Orgs.)
QUESTÃO 44
Os Parâmetros Curriculares Nacionais (PCN) redirecionaram a Física, no Ensino
Médio, para estimular, nos alunos, o interesse por conhecer o mundo físico a partir
de procedimentos para formar cidadãos autônomos intelectualmente. Considerando
esse referencial, analise as seguintes abordagens presentes em materiais didáticos:
I - procedimentos de pesquisa de concepções de senso comum;
II - privilégio de aspectos teóricos;
III - utilização de novo saber em sua dimensão aplicada;
IV - apresentação do conhecimento como fruto da genialidade dos cientistas.
Para selecionar materiais didáticos que atendam às orientações dos PCN para o
Ensino Médio, devem ser consideradas APENAS as abordagens
(A)
I e III
(B)
I e IV
(C) II e III
(D) II e IV
(E)
III e IV
Gabarito: A
Tipo de questão: Múltipla Escolha
Resolutor: Prof. Dr. João Bernardes da Rocha Filho
Comentário:
O objetivo da questão é avaliar nos estudantes a capacidade de interpretar
uma diretriz dos PCNs, confrontando-a com possíveis abordagens em um trabalho
pedagógico, verificando quais delas são compatíveis com aquela diretriz.
Para responder essa pergunta é necessário saber que um dos objetivos do
ensino de Física é formar cidadãos autônomos intelectualmente. Essa autonomia
intelectual se manifesta, entre outras formas, pela capacidade de distinguir os
conhecimentos do senso comum dos conhecimentos científicos, pois assim os
cidadãos serão capazes de fazer escolhas adequadas, considerando um mesmo
problema sob diversos aspectos e sabendo, ainda, argumentar a favor ou contra
procedimentos ou posicionamentos, segundo seus pressupostos. Além disso, os
ENADE Comentado 2008: Física
77
cidadãos intelectualmente autônomos devem ser capazes de avaliar as implicações
da utilização do conhecimento científico, que surge na forma de produtos e processos
potencialmente danosos para o meio ambiente, por exemplo, evitando a manipulação
que o poder econômico pode exercer sobre a população via meios de comunicação.
Assim, podemos dizer que o:
1. O item I cita uma abordagem útil ao alcance da diretriz dos PCNs citada no
enunciado da questão, pois com base nela o estudante é estimulado a
instrumentalizar-se para compreender a ciência e o pensamento científico,
confrontando-o com o senso comum, o que o auxilia a atingir a autonomia
intelectual desejada;
2. O item II é útil para a formação de cientistas, porém tem pouca relação com a
formação de cidadãos autônomos ou com o estímulo para que os alunos
conheçam o mundo físico. Ao contrário, é frequente que a ênfase nos
aspectos teóricos do conhecimento físico seja compreendida pelos estudantes
do nível médio como uma barreira à aprendizagem da Física, o que os afasta
da ciência e entrava o desenvolvimento da autonomia que se deseja.
3. O item III se relaciona com a contextualização no ensino de Física, que é
um recurso pedagógico que contribui para formar cidadãos capazes de
avaliar a pertinência, a utilidade e a nocividade de novos produtos e
tecnologias,
levando-os,
posteriormente,
ao
desenvolvimento
da
capacidade de tomar partido, por exemplo, quanto à pertinência ou não do
financiamento público de pesquisas científicas para um dado tema, que é
um dos papéis esperados de um cidadão completo.
4. O item IV é um equívoco baseado em um erro epistemológico porque mais
afasta os estudantes da ciência do que os aproxima dela, mostrando uma
ciência irreal, feita por gênios excêntricos. Nem isso é verdade, já que os
cientistas são pessoas comuns, que têm famílias e vivem em sociedade,
nem essa estória romanceada e permeada de gênios da ciência ajuda a
despertar novas vocações científicas, pois produz um rebaixamento da
autoestima dos estudantes.
Sendo assim, as afirmações corretas são a I e a III, portanto a alternativa
correta é a letra (A).
78
Maria Eulália Pinto Tarragó e Délcio Basso (Orgs.)
QUESTÃO 45
Calor e temperatura são conceitos estatísticos ligados às propriedades coletivas das
partículas que constituem os corpos: a temperatura está ligada à energia cinética
média das partículas e o calor, às trocas de energia entre os constituintes dos
corpos. Ao utilizar em aula um termoscópio, o professor, associando discussões
históricas ao experimento, possibilitará que seus alunos distingam os conceitos de
temperatura e calor, ao constatarem que, quando ele segura o termoscópio, o nível
do líquido
(A)
aumenta, caso a temperatura do professor seja superior à do ambiente.
(B)
aumenta, caso a temperatura do professor seja igual à do ambiente.
(C) aumenta, para qualquer temperatura ambiente.
(D) não se altera, caso a temperatura do professor seja menor que a do ambiente.
(E)
diminui, caso a temperatura do professor seja maior que a do ambiente.
Gabarito: A
Tipo de questão: Múltipla Escolha
Resolutor: Prof. Dr. João Bernardes da Rocha Filho
Comentário:
Essa questão objetiva verificar se o estudante compreende o que é um
termoscópio e se é capaz de utilizar a lei do equilíbrio termodinâmico em uma
situação experimental, além de ter conhecimento sobre a dilatação térmica. O
termoscópio é uma espécie de termômetro de líquido em vidro podendo conter dois
bulbos e um tubo, ou um bulbo e um tubo, contendo líquido e ar em seu interior.
Existem muitas variações modernas do termoscópio, porém vamos fixar-nos, nessa
análise, no termoscópio galileano, que era aberto na parte superior e usava água, e
no termoscópio de Médici, que era fechado na parte superior, e usava álcool.
Vamos analisar, uma a uma, as alternativas, propostas a partir da afirmação
de que o professor segura o termoscópio, assumindo que o termoscópio esteja em
equilíbrio termodinâmico com o ambiente antes do professor segurá-lo:
ENADE Comentado 2008: Física
79
(A) essa alternativa propõe que o nível do líquido no instrumento aumente
caso a temperatura do professor seja maior do que a do ambiente. Essa é
uma alternativa correta, pois o líquido e o ar do bulbo inferior,
inicialmente em temperatura ambiente, serão aquecidos e dilatarão mais
que o vidro do recipiente. Com isso, a pressão no interior do bulbo inferior
aumentará em relação à pressão atmosférica ou à pressão existente no
bulbo superior, ainda frio, empurrando para cima o líquido pelo tubo
vertical (aberto ou fechado, conforme o tipo de termoscópio);
(B) essa alternativa está errada, pois se não há diferença de temperatura
entre a mão do professor e dos materiais do termoscópio, não haverá
variação da pressão interna, e nenhum líquido será empurrado através do
tubo vertical;
(C) essa alternativa está errada, pois a temperatura ambiente influencia
diretamente o funcionamento do termoscópio, como vimos acima;
(D) essa alternativa está errada, pois o líquido deveria baixar no tubo vertical,
já que a pressão interna do termoscópio iria diminuir com o resfriamento
causado pela mão do professor;
(E) essa alternativa está errada, pois é precisamente o oposto do que foi
afirmado no item (A).
80
Maria Eulália Pinto Tarragó e Délcio Basso (Orgs.)
QUESTÃO 46 (QUESTÃO ANULADA)
Com relação à produção e à utilização das Tecnologias da Informação e da
Comunicação (TIC), pode-se afirmar que:
I - a TIC deve adaptar-se às necessidades de determinado projeto políticopedagógico;
II - a introdução de novas TIC na educação implica novas práticas pedagógicas;
III - o planejamento das TIC deve permitir a reflexão dos estudantes sobre diferentes
abordagens de um mesmo problema.
Está(ão) correta(s) a(s) afirmação(ões)
(A)
I, apenas.
(B)
II, apenas.
(C) I e III, apenas.
(D) II e III, apenas.
(E)
I, II e III.
Gabarito: C
Tipo de questão: Múltipla Escolha
Resolutor: Prof. Dr. Aldoir Rigoni
Comentário:
Essa questão trata da produção e uso das Tecnologias da Informação e
Comunicação (TIC) nos processos educacionais e de formação profissional nas
Instituições de Ensino. As novas tecnologias educacionais estão presentes nas
Instituições de Ensino em maior ou menor intensidade e podemos dizer que são
imprescindíveis. As TIC são itens de verificação das Comissões de Especialistas do
MEC, tanto na avaliação para o reconhecimento de cursos como nas verificações
das condições de ensino. O propósito da questão é o de verificar em que grau e
como o estudante percebe e acompanha a presença desses novos recursos
tecnológicos na ação didático-pedagógica. Recursos estes constantes nos Projeto
Pedagógico do Curso – PPC.
ENADE Comentado 2008: Física
81
Em relação às alternativas de resposta à questão formulada podem ser feitas
as seguintes considerações:
a) a alternativa I está correta, pois a Tecnologia da Informação e
Comunicação deve estar em consonância com as necessidades previstas no Projeto
Pedagógico do Curso – PPC, que na sua construção prevê os recursos tecnológicos
necessários ao desenvolvimento do ensino.
b) A alternativa II não é procedente, pois as novas Tecnologias estão a
serviço das práticas pedagógicas e não necessariamente determinam novas
práticas.
c) A alternativa III está correta, pois um mesmo problema sempre deve
permitir variadas abordagens, o que é feito, via de regra, com o uso de diferentes
tecnologias de ensino, facilitando a reflexão dos estudantes sobre o mesmo assunto
com o envolvimento de uma quantidade maior de faculdades mentais no processo.
Assim a resposta correta é a letra (C).
82
Maria Eulália Pinto Tarragó e Délcio Basso (Orgs.)
QUESTÃO 47
O desenvolvimento da ação educativa a partir da construção de projetos político-pedagógicos tornou-se uma obrigação para as escolas. Neste contexto, a Física
deve participar, aproveitando os momentos pedagógicos para trabalhar seus
conteúdos. Numa escola hipotética, a realização de uma peça teatral é um desses
momentos. Os professores de Física resolvem trabalhar, assim, com o tema
estruturador 4 dos PCN+: Som, Imagem e Informação. O projeto da Física trabalhará
com a iluminação do palco, sem esquecer as diretrizes do cenógrafo. As
características do teatro são: as paredes do palco, quando iluminadas com luz verde
e vermelho, misturadas, ficam amarelas, e o piso, quando iluminado com luz verde e
azul, misturadas, fica ciano. As cores das paredes do palco e do piso,
respectivamente, são:
(A)
amarelo e azul.
(B)
amarelo e amarelo.
(C) verde e magenta.
(D) azul e branco.
(E)
branco e branco.
Gabarito: E
Tipo de questão: Múltipla Escolha
Resolutor: Prof. Dr. Aldoir Rigoni
Comentário:
Na resolução dessa questão o estudante deve saber que a luz branca forma-se da composição de todas as cores do espectro. Uma superfície branca tem a
propriedade de refletir todos os raios luminosos que nela incidem.
A cor amarela é uma cor secundária resultante da sobreposição das cores
vermelha e verde. A incidência das cores vermelho e verde em paredes brancas as
tornam amarelas, pois esta é a composição resultante da mistura. Assim, as paredes
do palco devem ser brancas.
O mesmo raciocínio pode ser feito em relação à cor ciano, resultante da
mistura das luzes azul e verde, portanto o palco também deve ter cor branca. Assim,
considerando-se o que foi exposto, conclui-se que a alternativa correta é a letra (E).
ENADE Comentado 2008: Física
83
QUESTÃO 48
No vocabulário pedagógico do MEC, presente nos Parâmetros Curriculares
Nacionais (PCN), interdisciplinaridade, contextualização e autonomia são três pilares
fundamentais da Educação. Nessa perspectiva, procurando seguir as orientações
oficiais dos PCN, os currículos escolares apresentam algumas das recomendações
abaixo.
I - A interdisciplinaridade não deve preceder a disciplinaridade.
II - Uma referência fundamental é considerar o que o jovem precisa para viver em
um mundo tecnológico complexo e em transformação.
III - As disciplinas afins devem ser agrupadas em uma única disciplina.
IV - A lista de tópicos dos programas não deve ser o foco principal.
Estão de acordo com as orientações oficiais APENAS os currículos que seguem as
recomendações
(A)
I e II
(B)
I, II e III
(C) I, II e IV
(D) I, III e IV
(E)
II, III e IV
Gabarito: Questão C
Tipo de questão: Múltipla Escolha
Resolutor: Prof. Dr. Aldoir Rigoni
Comentário:
Autonomia, contextualização e interdisciplinaridade formam o tripé de uma
nova maneira de ver a educação adaptada aos nossos tempos.
A
organização
e
tratamento
dos
conteúdos
devem
contemplar
a
interdisciplinaridade que garante o inter-relacionamento das disciplinas em áreas
específicas. É notório que a interdisciplinaridade é posterior à definição das
disciplinas, pois aquela não subsiste sem estas. A interdisciplinaridade permite o
aperfeiçoamento das estruturas curriculares nas escolas.
84
Maria Eulália Pinto Tarragó e Délcio Basso (Orgs.)
A contextualização relaciona de maneira perceptível o conteúdo ensinado nas
escolas com os âmbitos ou dimensões presentes na vida pessoal, social e cultural
dos alunos. Incentiva comportamento ativo e participante na busca do que precisam
para viver num mundo competitivo, tecnológico e de permanente transformação. Por
falta de contextualização muitas pessoas que estudaram física na escola talvez não
consigam entender como funciona um celular, um controle eletrônico ou não saibam
estabelecer relações entre o aquecimento de água e o consumo de energia elétrica,
ou de gás.
Não necessariamente as disciplinas afins devem ser reunidas em uma única,
constituindo um eixo ou bloco de ensino. Essa ação pode acarretar dificuldades de
ordem
acadêmico-administrativas,
relacionadas
aos
créditos
e
ao
grande
entrosamento pedagógico que deveria ocorrer entre os professores que fossem
ministrar o conjunto de disciplinas reunidas.
De outra parte procede afirmar que a lista de tópicos dos programas não deve
ser o principal, uma vez que a escolha dos tópicos é necessariamente precedida
pelos objetivos e pelo perfil profissional definidos nos Projetos Pedagógicos dos
Cursos.
A partir das considerações feitas é possível concluir que somente o item III
não está correto, estando todas as demais recomendações de acordo com as
orientações oficiais. Logo, a alternativa (C) é a correta.
ENADE Comentado 2008: Física
85
QUESTÃO 49 - DISCURSIVA
Reconhecendo que os sistemas democráticos se tornam vulneráveis sem a cultura
científica, um professor de Física concordou com as preocupações expressas nos
Parâmetros Curriculares Nacionais (PCN) sobre a formação do cidadão e com as
sugestões de mudanças curriculares a serem adotadas nas escolas.
Nessa perspectiva, levando em conta os aspectos contextualizadores no cotidiano e
na História da Ciência e os aspectos epistemológicos e metodológicos do Ensino da
Física, descreva uma atividade a ser realizada em uma Unidade de Ensino sendo o
tema estruturador “Universo, Terra e Vida”, para cada habilidade abaixo.
a) Adquirir uma “compreensão atualizada das hipóteses, modelos e formas de
investigação sobre a origem e evolução do Universo”.
(valor: 5,0 pontos)
b) Identificar formas pelas quais os modelos explicativos do Universo influenciaram
a cultura e a vida humana ao longo da história da humanidade e vice-versa.
(valor: 5,0 pontos)
Tipo de questão: Discursiva
Resolutores: Profª. Dr. Maria Eulália Pinto Tarragó e Prof. Me. Délcio Basso
Comentário:
a) Atividades propostas para desenvolver a “compreensão atualizada das
hipóteses, modelos e formas de investigação sobre a origem e evolução do
Universo”:
Leitura de capítulos de livros, visita a sítios na internet e projeção de filmes –
seguidos de discussão – que tratem sobre o tema, como, por exemplo:
Livros:
• Descobrindo o Universo, Neil F. Comins & William J. Kaufmann III, Ed.
Bookmann. 2010
• Guia Ilustrado Zahar: Astronomia. Ian Ridpath. Jorge Zahar Editor.
Filmes:
• “De olho no Céu”, produzido e divulgado pela IAU, no Ano Internacional da
Astronomia (2009).
86
Maria Eulália Pinto Tarragó e Délcio Basso (Orgs.)
• Série Cosmos. Carl Sagan. Editora Abril.
• The Universe. The History Channel.
Sítios na internet (sites):
http://www.telescopiosastronomicos.com.br/
http://www.pucrs.br/fisica/astronomia
http://cdcc.sc.usp.br/cda/aprendendo-basico/universo/universo.htm
Ao final dessas atividades o aluno deverá elaborar um texto que, na sua
opinião, destaque o que lhe pareceu mais significativo.
b) Formas como os modelos explicativos do universo influenciaram a vida humana e
a cultura ao longo da história da humanidade.
Ao longo do tempo, a observação do Universo, quer a olho nu ou com
telescópio, acabou levando a mudanças nas concepções cosmológicas e nas físicas
que lhe dão suporte. Por exemplo:
A cosmologia e a física aristotélicas acabaram sendo substituídas por uma
cosmologia heliocêntrica. Essa nova concepção cosmológica de colocar o Sol no
centro e a Terra como planeta gerou uma nova física, com as Leis do Movimento e a
Lei da Gravitação, a qual serviu de ponto de partida para o desenvolvimento de
outras áreas da física como a termodinâmica e o eletromagnetismo. Estas, por sua
vez, levaram ao aperfeiçoamento das máquinas térmicas que propiciaram a
Revolução Industrial e ao surgimento da era da Eletricidade, que levou até a era das
telecomunicações e da telemática, era que estamos vivendo atualmente.
ENADE Comentado 2008: Física
87
QUESTÃO 50 - DISCURSIVA
Nos circuitos das Figuras 1 e 2 abaixo, as pilhas e as lâmpadas são idênticas. Ao
prever o brilho da lâmpada L1 em relação aos brilhos das lâmpadas L2 e L3, nos
dois circuitos, é muito comum que alunos do Ensino Médio apresentem concepções
alternativas às concepções científicas.
a) A esse respeito, apresente uma concepção científica e uma possível concepção
alternativa, com a justificativa que os alunos poderiam apresentar.
(valor: 5,0 pontos)
b) Descreva uma estratégia de ensino contextualizada para que os alunos avancem
em direção ao conhecimento científico, realizando aprendizagem significativa dos
conceitos de corrente elétrica, resistência elétrica, resistência equivalente e
diferença de potencial. Indique nessa estratégia como o mundo vivencial dos
alunos e as relações de Ciência, Tecnologia e Sociedade (CT&S) podem ser
considerados e os recursos metodológicos a serem utilizados.
(valor: 5,0 pontos)
Tipo de questão: Discursiva
Resolutores: Adriana Schier e Profª. Dr. Sayonara Cabral da Costa
Comentário:
Concepção científica: inicialmente as três lâmpadas estão submetidas à
mesma diferença de potencial V do gerador, pois no 2° circuito L2 e L3 estão ligados
em paralelo com o gerador e no 1° circuito L1 é a única lâmpada. Então observando
88
Maria Eulália Pinto Tarragó e Délcio Basso (Orgs.)
os desenhos, não ocorre mudança no brilho das lâmpadas, mesmo se uma ligação é
em série e a outra em paralelo.
Concepção alternativa: uma grande parte dos alunos acredita que na figura 2
as lâmpadas teriam um brilho menor, “como se a energia fosse dividida por dois[...]”
Apesar de brilharem igualmente entre si, L1 teria um brilho maior do que o de L2 e
L3. Mas L2 e L3 brilham igualmente.
Como R2 = R3 = R
1/Req = 1/R2 + 1/R3 = 2/R
então Req = R/2
Com essa demonstração matemática podemos observar que a resistência
equivalente irá diminuir e a corrente ira aumentar.
A corrente elétrica na figura 1(a partir de V = iR):
V = i . Req
i = V/R
Corrente elétrica na figura 2:
i= V
i = 2V/R
R/2
A energia do gerador é P = Ri2, os alunos não se dão conta de que a corrente
na figura 1 é a mesma da figura 2. Em uma associação em paralelo, a corrente
aumenta, a resistência diminui e o brilho das lâmpadas é igual.
Resposta para item (b):
Para começar, a estratégia de ensino que pode ser utilizada, é a visualização
(atividade experimental) das associações (mostrando série e paralelo), com fio de
ligação, pilhas e lâmpadas. Não explicando de início o fenômeno físico que ocorre,
logo irá despertar o interesse no aluno, porque a própria demonstração certifica que
sua concepção está um pouco equivocada, e a reação do aluno é de curiosidade.
Com isso, prendemos sua atenção para maiores explicações, que será o passo
seguinte.
ENADE Comentado 2008: Física
89
Na parte vivencial, a instalação elétrica de nossas casas é feita por
associação em paralelo. Nesse caso, é possível que todas as tomadas e ponto de
luz tenham a mesma diferença de potencial. O que vai mudar é a corrente que vai
circular em cada lâmpada, por cada dispositivo elétrico, a resistência das lâmpadas
é diferente e depende da sua potência, então quanto mais lâmpadas uma residência
possui, maior será a corrente elétrica que circula pela instalação.
Os cálculos matemáticos provêm de equações físicas simples, que podem ser
facilmente compreendidas.
90
Maria Eulália Pinto Tarragó e Délcio Basso (Orgs.)
LISTA DE CONTRIBUINTES
Adriana Schier
Adriano Moehlecke
Aldoir Rigoni
Alexandre Ferret
Aline Cristiane Pan
Artur Majolo Scheid
Cássio Stein Moura
Délcio Basso
Elaine Evani Streck
Elias Cantarelli Hoffmann
Izete Zanesco
Janaína Galho Borges
João Bernardes da Rocha Filho
Juliane Bernardes Marcolino
Maiara Oliveira Dalenogare
Márcio Galhardi
Maria do Carmo Baptista Lagreca
Maria Eulália Pinto Tarragó
Natthan Ruschel Soares
Rafael L. Zimmer
Ricardo Meurer Papaléo
Sayonara Salvador Cabral da Costa
ENADE Comentado 2008: Física
91
Download